General Aptitude Sheet Sloution - 2022
General Aptitude Sheet Sloution - 2022
Years Marks
2017(Set-1) 15
2017(Set-2) 15
2018 15
2019 15
2020 15
2021(Set-1) 15
2021(Set-2) 15
GENERAL APTITUDE GATE SYLLABUSVERBAL
APTITUDE
Narrative sequencing
QUANTITATIVE APTITUDE
Data interpretation: data graphs (bar graphs, pie charts, and other graphs representing
data), 2- and 3-dimensional plots, maps, andtables
ANALYTICAL APTITUDE
Analogy
SPATIAL APTITUDE
TABLE OF CONTENTS
1 QUANTITATIVE APTITUDE 1
2 SPATIAL APTITUDE 85
Q2. Ifthenumber5107k3204iscompletelydivisibleby9,thenthesmallestwholenumber in
place of k willbe
(a)2 (b)6 (c)5 (d)7
Answer: C
Q4. On dividing a certain number by 9342 we get 44 as remainder. If the same number is
divided by 18, what will be the remainder?
Answer: 8
Q7. What least number must be subtracted from 13601, so that the resultant number is
divisible by87?
Answer: 29
Solution: 13601 mod 87 = 29
Q8. [MSQ]
5349isaddedto3957.Then7062issubtractedfromthesum.Theresultis/aredivisible by
(a)4 (b)3 (c)7 (d)11
Answer: a, b, d
Solution: 3957 + 5349 – 7062 = 2244; which is not divisible by 7.
Q10. On multiplying a number by 7, the product is a number each of whose digits is 3. The
smallest such numberis
Answer: 47619
Solution: The smallest such number = 333333 /7 = 47619
Q12. [MSQ]
The difference of the squares of two consecutive odd integers is always divisible by
which of the following integers?
Q13. [MSQ]
If n is a natural number, then (6𝑛2+ 6𝑛) is always divisible by
(a)3 (b)12 (c)18 (d)24
Answer: a, b
Solution:(6𝑛2+6𝑛)=6𝑛(𝑛+1)=6×2𝑘=12𝑘;𝑤𝑒𝑟𝑒𝑘𝑖𝑠𝑎𝑛𝑎𝑡𝑢𝑟𝑎𝑙𝑛𝑢𝑚𝑏𝑒𝑟
So, it is divisible by both 3 and 12.
Q14. [MSQ]
If n is a natural number, then 4(𝑛3− 𝑛) is always divisible by
(a)3 (b)6 (c)12 (d)24
Answer: a, b, c, d
Solution:4(𝑛3−𝑛)=4(𝑛−1)𝑛(𝑛+1)=4×3!)𝑘= 24𝑘;𝑤𝑒𝑟𝑒𝑘𝑖𝑠𝑎𝑛𝑎𝑡𝑢𝑟𝑎𝑙𝑛𝑢𝑚𝑏𝑒𝑟
So, it is divisible by 3, 6, 12 and 24.
Q15. Whatwillbetheremainderwhen(6767+67)isdivisibleby68?
Q16. [MSQ]
Whichofthefollowingnumber(s)willcompletelydivide(4949−1)?
(a)12 (b)24 (c)48 (d) None of these
Answer: a, b,c
Q17. Whatwillbetheremainderwhen(172022+1)isdividedby288?
Answer: 2
Solution: (172022 + 1) 𝑚𝑜𝑑 288 = ((17)2)1011 + 1)𝑚𝑜𝑑 288
=(288+1)1011 mod288+1mod288=2
Q26. What is the largest number N, such that the number 210, 315 and 525 are divisible by
N?
Answer: 105
Q28. What is the largest number N, such that when the number 151, 193 and 424 are
divisiblebyNthenwegetsameremainderineachcase?
Answer: 21
Q30. What is the third largest number N, such that when the number 93, 128 and 199 are
divisible by N then we get remainder 3, 2 and 1respectively?
Answer: 6
Q32. Whatistheunitdigitof1111+1212+1313+1414+1515+166?
Answer: 7
Q36. [MSQ]
LetXbeanaturalnumbersuchthatX=465374−235374,thenwhichofthefollowing statement
is/aretrue?
(a) X mod 35 =0
(b) X mod 23 =0
(c) (𝑋 + 1)2mod 5 = 1
(d) 𝑋 mod 161 = 0
Answer: a, b, c, d
Q39. Whatwillbethesumofalldivisorsof16800?
Answer: 62496
Q40. Let H.C.F and L.C.M of 4/14, 6/21 and 8/42 are X and Y respectively, then
X/Y is equal to
(a)8 (b)24 (c)24 (d)1
7 147 72
Answer: D
Q41. Whatwillbetheunitdigitof2626(263)37(45)63?
Answer:0
Q42. Whatwillbetheunitdigitof2121+2222+2323+2424?
Answer:8
Q43. LetN=23253159whatistheremainderwhenNisdividedby17?
Answer: 4
Q44. The value of 9(999) mod 8is _
Answer: 1
Q45. Two numbers differ by 5. If their product is 336, then the sum of the two numbers
would be:
(a)41 (b)43 (c)37 (d)39
Answer: C
Q46. The multiplication of two numbers is 9375 and the quotient, when the greater is
divided by the smaller is 15. The sum of the numbers is:
(a) 600 (b)125 (c)275 (d)400
Answer: D
Q47. The product of the two prime numbers is 493. What will the L.C.M of these two
numbers?
(a)493 (b)17 (c)29 (d)Can‟t bedetermined
Answer: A
Q48. What is the remainder when: 1661 + 1551 + 1441 + 1331 + 1221 is divided by 20?
(a)25 (b)5 (c)11 (d)1
Answer: B
Q49. The sum of two numbers is 45 and their product is 500. The HCF of the numbers is:
(a)5 (b)9 (c)10 (d)15
Q52. Which of the following number which when divides 1265 leaves quotient 84 and
remainder 5?
(a)15 (b)18 (c)23 (d)61
Answer: A
Q53. A number gives a remainder 5 when it is divided by 8. What will be the remainder
when the square of the same number is divided by 4?
Q54. A hundred digit number is formed by writing first 54 natural numbers one after the
other as 123456…………….5354. What will be the remainder when this number is
divided by 8?
(a)1 (b)2 (c)4 (d)0
Answer: 2
Q55. Ifn=1+x,wherexistheproductoffourconsecutivepositiveintegers,thenwhichof
thefollowingstatementsis/aretrue?
Q56. Iftheunitdigitintheproduct(459×46×28*×484)is2,thedigitinplaceof*is:
Answer: 7
Q57. A person divided 100 rupees between his friends, if there will be 5 more friends than
each will get 1 rupee less. How many friend he had?
(a)20 (b)25 (c)30 (d)35
Percentage
Q58. 50% of a% of b is 75% of b% of C. Which of the following is C?
(a)1.5a (b)0.667a (c)0.5a (d)1.25a
Answer: B
Solution: 50% of a% of b = 75% of b% of C
0.5𝑎𝑏 0.75𝑏𝐶
⟹ =
100 100
⟹ 0.50𝑎 = 0.75𝐶
0.50
⟹𝐶= 𝑎 = 0.667𝑎
0.75
Q59. x% of y + y % of x =?
(a) 3%ofxy (b) 2%ofxy (c) 5%ofxy (d) None ofthese
Answer: B
Q60. If 90% of A = 30% of B and B= x% of A, then the value of x is
(a)800 (b)300 (c)700 (d) None ofthese
Answer: B
GENERAL APTITUDE PAGE 20
Q61. The price of cooking oil has increased by 15%. The percentage of reduction that a family
shouldeffectintheuseofcookingoil,soasnottoincreasetheexpenditureonthisaccount is
2 1 1
(a)15 % (b) 13 % (c) 17 % (d)None ofthese
23 23 23
Q68. The population of a town increases by 12% during first year and decreases by 10 %
during second year. If the present population is 50400, what it was 2 years ago?
(a)40000 (b)35000 (c)50000 (d) None ofthese
Answer: C
Answer: C
Q70. The salaries of A and B together is equal to Rs 10,000. A spends 80% of his salary and B
spends 70% of his salary. If now, their saving arethe same, what is A′s salary?
(a)6000 (b)5000 (c)4000 (d)7000
Answer: A
Q71. A man save 40% of his monthly salary. On account of an increase in price, he has to
increasehismonthlyexpenditureby40%,andisabletosaveRs40permonth.What is his
monthly salary?
(a)Rs1000 (b)Rs500 (c)Rs200 (d) Rs250
Answer: D
Q72. Duetofallinmanpower,productioninafactorydecreasesto40%.Bywhatpercentageshould
the working hours be increased to restore the original level ofproduction?
(a)662% (b)461% (c)25% (d)40 %
3 3
Answer: A
Q73. Afterspending20%onmachinery,10%onbuliding,9%onrawmaterialsand7%on other,
Chandra had a balance of Rs. 2700, then the money with him initially was.
(a)Rs5000 (b)Rs6000 (c)Rs7000 (d) Rs8000
Answer: A
Q74. The population of a city decreases by 10% at the end of first year and increases by 10
at the end of second year and again decreases by 10% at the end of the thridyear .If
thepopulationofthecityattheendofthirdyearis4,455,thenwhatwasthepopulation
ofthecityatthebeginningofthefirstyers?
(a)5000 (b)4500 (c)4950 (d)1000
Answer: C
Q77. Theproductionofricewentupfrom2000milliontonnesin1998to2300milliontonnes
in1999.Whatisthepercentageincreaseintheproductionofriceform1998to1999? (a)20%
(b)15% (c)25% (d)12½%
Answer: B
Q78. In an examination 35% of students passed in maths, 48% passes in science and 23%
passed in both the subject . What is the percentage of the student who failed in both
the subject?
(a)50% (b)40% (c)60% (d)53%
Answer: B
Q79. In an election contested by two candidates. The candidates who gets 43% is rejected
by a margin of 2856 votes. What is the total number of votes polled given that all the
votes polled were valid votes?
(a)20,400 (b)20,820 (c)16,500 (d)40,800
Answer: A
Q80. In an election a candidate secures 37% of the total votes polled. The only
othercandidate defeated him by 520 votes. If all the votes polled are valid, the
number ofvotes polledwas
(a)1500 (b)4000 (c)2000 (d)4400
Answer: C
Answer: C
Q82. A person buys a horse for 15 pounds. After one year, he sells it for 20 pounds. After
one year, again he buys the same horse at 30 pounds and sells it for 40 pounds. What
is the overall profit percent for that person over both the transactions?
(a)30.33% (b)33.33% (c)35.33% (d)40.33%
Answer: B
Q83. Atradersells85mofclothforRs.8,925attheprofitofRs.15/mofcloth.Whatisthe cost price
of 1 m ofcloth?
(a)Rs.84 (b)Rs.90 (c) Rs.96 (d) Rs.102
Answer: B
Q84. By selling an article at Rs. 800, a shopkeeper makes a profit of 25%. At what price
should he sell the article so as to make a loss of 25%?
(a)Rs.460 (b)Rs.480 (c)Rs. 500 (d) Rs.520
Answer: B
Q85. Ankit bought 160 shirts at the rate of Rs. 225/shirt. The transport expenditure wasRs.
1,400. He paid an octroi at the rate of Rs. 1.75/shirt and labour charges were Rs. 320.
Whatshouldbethesellingpriceof1shirt,ifhewantsaprofitof20%?
(a)Rs.260 (b)Rs.275 (c) Rs. 280 (d) Rs.285
Answer: D
Q86. A man sold two houses for Rs. 8 lakhs each. On the one, he gained 6% and on the
other, he lost 6%. What percent is the effect of the sale on the whole?
(a) 0.36%loss (b) 0.36%gain (c) 36%loss (d) 36 %gain
Answer: A
Q87. If two articles are bought for the same price (i.e., the cost prices are equal) and one is
sold at a profit of 30% and the second is sold at a profit of 20%, then the overall
percentage of profitis
Answer: 25
Answer:B
Q97. Thecostpriceof30lemonsisthesameassellingpriceof24lemons.Whatispercentage
ofgain/loss?
(a) 25%profit (b)20%profit (c) 25%loss (d) 20%
profitAnswer:A
Q98. Thecostpriceofacertainnumberofarticlesisthesameasthesellingpriceof1/3𝑟𝑑
of the same number of articles. What is the gain/loss percent?
(a)66.67%gain (b) 200%gain (c)33%gain (d) 66.67%loss
Answer:B
Q99. A dealer weighs only 950 gm per kg. What is his error percentage?
5
(a) 5 % (b)5% (c)15% (d) None ofthese
19
Answer: B
Q100. A dealer professes to sell his goods at cost price and uses an 880 gm weight instead of
a kg. What is his percentage of gain? (Approximately)
(a)13.13% (b) 13.33% (c)13.36% (d) 13.64%
Answer: D
Q101. Atradergainsthecostof2kgofoilbyselling10kgofoil.Whatishisgainpercentage?
2 2
(a)20% (b)25% (c)18 % (d) 11 %
3 3
Answer: A
Q102. Aftertwosuccessivediscountsof10%eachareallowed,anarticleissoldat Rs
2,430. What is the discountallowed?
(a)Rs510.30 (b)Rs570 (c)Rs486 (d) Rs461.70
Q105. If30%and50%aretwosuccessivediscounts,thenitisequivalenttoasinglediscount
percentage , which is equalto
(a)80% (b)40% (c)65% (d)60%
Answer: C
Q106. The listed price of a bag is Rs 160. A merchant bought it for Rs 122.40. He got two
successive discounts of which one is 10%. What is other discount?
(a)10% (b)15% (c)20% (d)25%
Answer: B
Q107. The sale price of a „T‟ shirt, listed for Rs 400, after successive discounts 10% and 5%, is
(a)Rs340 (b)Rs341 (c)Rs342 (d) Rs343
Answer: C
Q108. XsellsYanarticleataprofitof20%.YsellsittoZatalossof20%.IfZpaidRs960,
thenwhatisthesellingpriceofX?
Answer: 1000
Q109. Ifthepriceofanappleisreducedby20%,apersoncanbuy5applesmoreforRs100,
whatisthereducedcostofeachapple?
(a)Rs5 (b)Rs4 (c)Rs1 (d)
Rs4.80Answer:B
CI and SI
Q111. Whatistheinterest(inRs)onasumofRs80,000atarateof20%peranumforthree years under
simpleinterest?
Answer: 48000
Solution: Simple interest = 80,000 × 0.2 × 3 = 48000 Rs.
Q112. A sum triples in 16 years at simple interest. What is the rate of interest per anum?
(a)10% (b)5.33% (c)12.5% (d) none ofthese
Answer: C
Solution:Rateofinterest=200×100=12.5%
16×100
Q117. A sum of money invested at compound triples itself in four years. In how many years
will it becomes 27 times itself at the same rate of compound interest?
(a)1.33years (b)8years (c)12years (d) 20
yearsAnswer:C
Q118. AsumofmoneyamountstoRs36,000inthreeyearsandRs48000infouryearsat
compoundinterest,compoundedannuallyineachcase.Whatistherateofinterestper
annum?
(a)66.67% (b)33.33% (c)20% (d)25%
Answer: B
Q119. There is 60% increase in an amount in 6 years at simple interest. What will be the
compound interest of Rs. 12,000 after 3 years at the same rate?
(a)Rs.2160 (b)Rs.3120 (c) Rs.3972 (d) Rs.6240
Answer: C
Solution:
Q121. Asumofmoneybecomes16timesofitselfin2yearsifcompoundedhalf-yearly.How
muchtimeitwilltaketobecome27timesifcompoundedyearly.
(a)3years (b)4years (c) 5years (d) 6years
Answer: A
Solution:
Q123. If the ratio of difference of S.I and C.I. for two years and 3 years is 4:13. Find the rate
of interest.
(a)20% (b)25% (c)30% (d) 40%
Answer: B
Solution:
The ratio of difference of S.I and C.I. for two years and 3 years is
𝑟 2 𝑟 2 𝑟
𝑃( ):𝑃( )(3+ )=100:(300+𝑟)100100 100
But in question, it is given that 4 : 13.
Therefore,100:(300+𝑟)=4∶13
So, r = 25.
Q124. The difference between SI and CI for 2 years at the rate of 20% per annum is Rs 16.
What is the principal value?
(a)Rs.200 (b)Rs.400 (c)Rs. 500 (d) Rs.800
Answer: B
Solution: Using the formula: Difference = P (R/100) 2
16 = P[20/100]2
On Solving, P = Rs 400
Q127. ThedifferencebetweenthesimpleinterestandcompoundinterestonasumofRs.5,00,000
fortwoyearsisRs2450.Whatistherateofinterest,ifitiscompoundedannually
(a)6% (b)7% (c)8% (d)10%
Answer: B
Q128. AsumofRs.7,500investedatcompoundinterestdoublesinfiveyears.After20years
itbecomes.
(a)Rs1,50,000 (b)Rs60,000 (c)Rs 1,20,000 (d)Rs 90000
Answer: C
Q129. In how many years will a sum of money , when invested at 25% p.a. Compound
interest compounded annually, becomes 1.5625 times (approximately one and half
times) itself ?
(a)1 (b)2 (c)3 (d)4
Answer: B
Q130. A sum amounts to Rs 8640 in three years at 20 % per anum compound interest. What
was the value of the sum?
(a)Rs7500 (b)Rs5000 (c)Rs 6000 (d)Rs4500
Answer: B
Q131. Findthedifferenceinthecompoundinterestat40%p.awhenitiscompoundonRs
20,000foroneyear,whentheinterestiscompoundedhalfyearlyandquarterly.
(a)Rs284 (b)Rs364 (c)Rs 482 (d) Rs436
Answer: C
Answer: B
Q133. ThedifferenceincompoundinterestaccruedonRs20,000foroneyearat6%p/awhen
interestiscompoundedyearlyandwhenitiscompoundedhalfyearlyis
(a)Rs32 (b)Rs15 (c)Rs 18 (d)Rs21
Answer: C
Q134. The difference between the simple interests received from two different banks on a
sumofRs60,000fortwoyearsisRs1,500.Whatisthedifferenceoftheinterestrates of the
two banks in percentagepoints?
(a)1.25 (b)1.5 (c)0.5 (d)0.75
Answer: A
Q135. If the difference of the simple interest and the compound interest on a certain sum at
15% p.a. for two years is Rs 360. Find the sum.
(a)Rs2,400 (b)Rs20,000 (c) Rs 16,000 (d) Rs4.800
Answer: C
Q136. Ifthesimpleinterestonasumofmoneyat10%p.aforfouryearsisRs2,400thenthe
compound interest on the same sum for the same period at the same rate, when
interest is compounded annually,is
(a)Rs 2847.60 (b)7248.80 (c)2784.60 (d)Rs4284.70
Answer: C
Q137. A sum of money doubles itself at a certain rate of compound interest, compounded
annually in six years. In how many years (approximately) will it triple itself at the
same rate of compound interest? (log 3: log2 = 1.5850)
Answer: B
Q139. Hari gave a loan of Rs 20 to Ram and recover at a rate of Rs 3.50 for eight months,
commencingfromtheendof1stmonth.Whatistheeffectiverateofsimpleinterestper
annum?
(a)60% (b)80% (c)40% (d)90%
Answer: A
Q140. A sum of Rs 800 amount to Rs 1,040 in four years at simple interest. By how many
percentage points should the rate of interest per annum be increased so that Rs 1,200
would amount to rs 2,400 in 8 years?
(a)4 (b)8 (c)5 (d)None of these.
Answer:C
Q141. The effective annual rate of interest corresponding to a nominal rate of 6% per annum
payable half-yearly is:
(a)6.06% (b)6.07% (c)6.08% (d)6.09%
Answer: D
Solution:Effectiverateofinterest=100×[(1+0.03)2−1]= 6.09%
Q142. JohninvestsRs.5,000inatermdepositscheme.Theschemeoffersaninterestrateof
6%perannum,compoundedquarterly.Whatistheeffectiverateofinterest?
(a)0.06% (b)61.3% (c)6.13% (d)6%
Answer: C
Solution: We know that,
Principalamount=P=Rs.5,000
Actual rate of interest = i = 6% p.a. = 0.06 p.a. = 0.015 per quarter
Q145. The expenditure and savings of a person are in the ratio of 4:1. If his savings are
increased by 25% of his income, then what is the new ratio of his expenditureand
savings?
(a)8:5 (b)7:5 (c)16:9 (d)11:9
Q146. A bag containing twelve goggles is dropped from the first floor of a building. Which
of the following cannot be the ratio of the number of goggles broken to unbroken?
(a)1:5 (b)5:7 (c)2:1 (d)4:3
Q147. The ratio of marks obtained by Ram and Das is 4 : 3. If Ram and Das get 210 marks
together and the average of their percentage is 70, find their individual marks and the
total marks for which the exam was conducted.
(a)160,120,200
(b)180,135,200
(c)150,120,200
(d)120,90,150
Q149. Find the three numbers in the ratio 6:8:3 such that the difference of the square of the
first and the third number is 108.
(a)6,8,3 (b)18,24,9 (c)9,12,4 (d)12,16,6
Q150. The ratio of A‟s salary to B‟s was 4 : 5. A‟s salary is increased by 10% and B‟s by 20%,
what is the ratio of their salaries now?
(a)14:11 (b)15:14 (c)11:15 (d) None ofthese
Answer: C
Solution: Present ratio = 4 : 5. Increase of 10% and 20%.
New ratio of salaries will be 4 × 1.1 : 5 × 1.2 = 11 : 15.
Q151. 300 coins consists of 1 rupee, 50 paise and 25 paise coins, their values being in the ratio
of 10 : 4 : 3. Find the number of coins of each type.
(a) 100,80,120 (b) 80,90,100 (c) 100,100,80 (d) 60, 80,100
Answer: A
Solution:
Value of rupee coins = Rs10 i.e. 10 coins.
Value of 50 p coins = Rs 4 i.e. 8 coins.
Value of 25 p coins = Rs. 3 i.e. 12 coins.
∴ Ratio of coins = 10 : 8 : 12 ⇒ 5 : 4 : 6.
∴Numberofrupeecoins=5/15×300=100.
Numberof50Pcoins=4/15×300=80and
Numberof25Pcoins=6/15×300=120
Q152. Ifxvariesinverselyas(y2–1)andisequalto24wheny=10,thenthevalueofxwhen
y=5willbe?
Answer: 99
Q153. Rs.590 is divided amongst A, B, C so that 5 times A's share, six times B's share and
eight times C's share are all equal. What is C's share?
Q154. The salary earned by a man for three months in the ratio of 2 : 4 : 5. If the difference
between the product of salaries of the first two months to that of the last two months
(in Rs) is 4,80, 00, 000, then what is the salary of the man for the second month?
Answer: 8000
Q155. Theforceofattractionbetweentwobodiesvariesinverslyasthesquareofthedistance
betweenthem.Whenthedistanceis20feet,theforceofattractionis18N.Whatisthe
forceofattraction,whenthedistanceis30feet?
(a)8N (b)20N (c)40N (d) 80N
Answer: A
Q156. The length of the edges of two cubes are in the ratio 5 : 4. What is the ratio of their
diagonal?
Q161. A, B, C started a business with their investments in the ratio 1:3:5. After 4 months, A
invested the same amount as before and B as well as C withdrew half of their
investments. The ratio of their profits at the end of the year is :
(a) 1 : 2:3 (b) 3 : 4:15 (c) 3 : 5:10 (d) 5 : 6 :10
Answer: D
Solution: Let their initial investments be x, 3x and 5x respectively. Then,
A:B:C = (x*4+2x*8) : (3x*4+(3x/2)*8) : (5x*4+(5x/2)*8)
20x : 24x : 40x = 5 : 6 : 10
Q162. A, B, C rent a pasture. A puts 10 oxen for 7 months, B puts 12 oxen for 5 months and
Cputs15oxenfor3monthsforgrazing.IftherentofthepastureisRs.175,howmuch must C
pay as his share ofrent?
(a)Rs.45 (b)Rs.50 (c)Rs.55 (d) Rs.60
Answer: A
Solution:
A:B:C = (10×7):(12×5):(15×3)
= 70 :60 :45
= 14 :12 :9
Q164. A, B and C enter into a partnership and their shares are in the ratio 1/2 : 1/3 : 1/4.
After2months,Awithdrawshalfofhiscapitalandafter10months,aprofitofRs.378 is
divided among them. What is B'sshare?
(a)144 (b)169 (c)225 (d)339
Answer: A
Solution:
Ratio of initial investments = 1/2 : 1/3 : 1/4 = 6 : 4 : 3.
Lettheirinitialinvestmentsbe6x,2xand3xrespectively.
Q165. A invested Rs x for 5 months and Rs y for the remaining period. B invested Rs y for
thefirst8monthsandRsxfortheremainingperiod.Ifattheendoftheyeartheyshare
theprofitsequally,thenwhatisthevaluex–y?
(a)x (b)2x (c)–y (d) 0
Answer:D
Q166. In a business, Ajay puts in Rs 10,000 and Karishma puts in Rs 20,000. Ajay received
20% of the profit for managing the business and the rest of the profit is divided in the
ratio of their capitals. What is Ajay‟s share of the profit in a profit of Rs 6000?
Answer: 2800
Q168. A, B, C invested their capitals in the ratio of 4 : 3 : 2. At the end of the year they shared
the profits in the ratio 8 : 9 : 8. Who among A, B and C stayed in the business for the
longest period?
(a)A (b)B (c)C (d) Can‟tsay
Q169. Kamal started business with Rs 6000 and was later joined by Bharat with Rs 8, 000.
After how many months did Bharat join, if the profit at the end of the year were
divided in the ratio of 3: 2?
(a)6 (b)8 (c)9 (d)10
Q170. A and B are partners in a business. If A stayed in the business for one year and
contributed1/3rdofthecapitalwhileBreceived3/5thoftheprofit,thenforhowlong
B‟smoneywasinthebusiness?
(a) 8months
(b) 6months
(c) 10months
(d) 9months
AVERAGES
Q171. Thereare50,60and40studentsinthreesectionsA,BandCofaclassrespectively.If the
average marks of the students in sections A, b and C are 65, 70 and 75 Respectively,
then find the approximate average marks of all three sections put together.
(a)70 (b)80 (c)60 (d)50
Answer: A
Answer: C
Q173. Theaverageageofagirlandherparentsis28years.Iftheratioofthepresentagesof
thefather,themotherandthegirlis7:6:1,thenwhatwillbegirl‟sageafter2years?
(a)7years (b)9years (c)6years (d) 8
yearsAnswer:D
Q174. The average of five numbers is 39.20 and the average of three of these numbers is 41.
What is the average of other two numbers?
(a) 35.5 (b)36.5 (c)37.5 (d)38.5
Answer: B
Q175. In a class of 60 students, the average weight of 30 students is x kg and that of the
remaining is y kg. What is the average weight of the class?
𝑥𝑦
(a)(x +y)kg (b)30kg (c)(𝑥+𝑦)kg (d)( )kg
2 2
Answer: C
Q176. Theaverageageoftheeightboys‟increasesbyoneyear,ifaboyof12yearsofageis
replacedbyanewboy.Whatistheageofthenewboy?
(a)9years (b)15 years (c)10years (d) 20
yearsAnswer:D
Q177. The average weight of A, B and C is 80 kg. If D joins, the average becomes 82 kg. If
anotherpersonE,whoseweightis3kglessthanD,replacesA,thentheaverageofB,
C,DandEbecomes78kg.WhatistheweightofA?
(a)101kg (b)92kg (c)85kg (d) 97kg
Answer: A
Q178. The average weight of 8 person's increases by 2.5 kg when a new person comes in
place of one of them weighing 65 kg. What might be the weight of the new person?
(a)76kg (b)76.5kg (c)85kg (d) Datainadequate
Q180. A car owner buys petrol at Rs.7.50, Rs. 8 and Rs. 8.50 per litre for three successive
years. What approximately is the average cost per litre of petrol if he spends Rs.4000
eachyear?
(a)Rs.7.98 (b)Rs.8 (c)Rs. 8.50 (d) Rs.9
Answer: A
Solution:
Q181. The average mark of 600 students in an examination is 42. Among them, the average
of the top 150 students is 76, while that of the last 250 students is 28. What are the
average marks of the remaining 200 students?
Q183. Among P, Q, R, S and T, P weighs 2 kg less than Q, Q weighs 2 kg more than R orS,
individually. But T weighs 6 kg more than Q. the average weight of these five will be
equaltotheweightofwhoamongthefollowing?
(a)P (b)Q (c)T (d)
RAnswer:B
Q184. Fouryearsago,theaverageageofAandBwas20years.IftodayaverageageofA,B
andCis25years,whatwillbeageofCafter7years?
(a)32 years (b)34years (c)36years (d)38years
Answer: B
Solution:TotalageofAandBfouryearsago=20∗2=40years Present
total age of A and B would be = 40 + 4∗2 = 48 years
PresenttotalageofA,BandCis=25∗3=75years
∴ Present age of C would be = 75 - 48 = 27 years
Age of C after 7 years = 27 + 7 = 34 years
Q185. Inanexam,theaveragemarkfor80studentsofClassVis35.Theaverageofmarks
insectionAoftheclassis55,whiletheaverageofmarksinsectionBis30.Findthe
Q187. If the average weight of a group of 30 people is 50 kg and another group of 40people
has an average weight of 60 kg, then the weighted average of two groups (i.e. , the
averageweightofthecombinedgroupof70people)is:
(a)55kg (b) 55.71kg (c)56kg (d) 53.5kg
Q190. How many kilogram of wheat at rs 9.30 per kg must be mixed with 16 kg of wheat at
Rs 13.80 per kg such that the mixture when sold at Rs 12.43 per kg gives a profit of
10%?
(a)24 (b)15 (c)20 (d)25
Answer: C
Q191. The ratio of copper and zinc in two alloys is 2: 3 and 3: 5 respectively. What is the
proportion of copper and zinc in an alloy obtained by mixing equal quantities ofthese
twoalloys?
(a) 21: 59 (b) 41: 39 (c) 31:49 (d) 69 : 11
Answer: C
Q192. In what ratio must two solutions containing beer and water in the ratios of
3 : 2 and 7 : 3 respectively be mixed such that the resultant solution has beer and water
in the ratio 2 :1?
(a) 1: 2 (b) 2: 3 (c) 3:2 (d) 2 :1
Answer: A
Q193. Theratioofthevolumesofmilkandwaterinasolutionis4:3.Byadding28litresof
water,theratiogetsreversedandthecanbecomesfull.Whatisthecapacityofthecan, inlitres?
(a)120 (b)112 (c)84 (d)76
Answer: B
Q194. A vessel contains a mixture of spirit and water with 40% spirit in it. When 10 litres of
pure spirit is added, the vessel becomes full and contains 50% spirit. What is the
capacity of vessel (in litres)?
Q196. A large tank has 49 litres of pure milk. Seven litres of milk is removed and replaced
with water. How many more times should this process repeated such that the resultant
solution has 30 6litres of milk?
7
Q198. The cost of Type 1 rice is Rs. 15 per kg and Type 2 rice is Rs. 20 per kg If both Type 1 and
Type 2 are mixed in the ratio of 2 : 3.then the price per kg of the mixed variety rice is :
(a)Rs.18 (b)Rs.18.50 (c)Rs.19 (d) Rs. 19.50
Answer: A
Q199. A container contains 40 litres of milk From this container 4 litres of milk was taken out
and replaced by water. This process was repeated further two times. How much milk is
Q201. Two vessels A and B contain milk and water mixed in the ratio of 8:5 and 5:2
respectively.Theratioinwhichthesetwomixturesbemixedtogetanewmixture
3
containing 69 % milk is :
13
Q202. 4/7 of 42 litres of a mixture of phenol and water is water. When k litres of water is added to
this mixture, the ratio of phenol and water becomes 2 : 3. What is the value of x?
Answer: C
Q203. Tea worth of Rs. 135/kg & Rs. 126/kg are mixed with a third variety in theratio 1 : 1 : 2. If
the mixture is worth Rs. 153 per kg, the price of the third variety per kg will be?
(a) Rs. 169.50 (b) Rs.1700 (c) Rs. 175.50 (d) Rs.180
Q204. A merchant has 1000 kg of sugar, part of which he sells at 8% profit and the rest at 18%
profit. He gains 14% on the whole. The quantity sold at 18% profit is: (a)400kg
Answer: C
Solution:
Q205. Eight men can do a work in 12 days. If there were two men less, in how many days
can they complete one-fourth of the originalwork?
Answer: 4
Q206. Acandoapieceofworkin20daysandBcandoitin30days.Inhowmanydayswill they
complete the worktogether?
Answer: 12
Q207. Forty eight clerks can clear 360 files in 15 days. How many clerks are needed toclear
480filesin12days?
Answer: 80
Q208. TheratioofthecapacitytodoaworkofAandBis3:2.Iftheytogethercancomplete
a work in 18 days, then how long does A take to complete the work alone?
Answer: D
Q210. A is four times as efficient as B while C is twice as efficient as B. if A can do a job in 6
days. How long will A, B and C together take to complete thework?
(a)31days (b) 33days (c) 33days (d)none ofthese
2 7 4
Q211. 4 men or 8 women or 12 boys can do a certain work in 98 days. How many days will
3 men, 5 women and 8 boys together take to do the work?
(a)40 (b)44 (c)46 (d) 48
Answer: D
Q212. Agroupof10boyscandoajobin12days.anothergroupof10girlscandothework in 6 days.
How many days are required to complete the work if both the group work together?
(a)8 (b)6 (c)4 (d) none of these
Q213. Two pipes A and B can fill a tank in 30 minutes and 40 minutes respectively. C can
empty the tank in 60 minutes. When the tank is empty all the three pipes are opened
for 12 minutes and then pipe B is closed. How many minutes, form the beginning,
does it take for the tank to be filled?
(a)42 (b)72 (c)24 (d)30
Q214. Xis50%moreefficientthanYindoingawork.ifthedifferenceoftheirearningsupon
completingtheworktogetherisRs160,howmuchdidthey receiveinall?
(a)Rs210 (b)Rs240 (c)Rs800 (d) Rs400
Q221. A can build a wall measuring 20 ft × 3 ft × 15 ft in 6 days. B can build a wall measuring
30ft x 4 ft x 10 ft in 5 days. What is the ratio of work done by A and B in on day?
(a) 8: 5 (b) 5: 8 (c) 6:7 (d) 7 :5
Q222. Acandigapitworkingeighthoursaday,Bcanfillitworkingtenhoursaday.Ifboth
ofthemstarttheworkatthesametime,afterhowmanydays,willtherebeafullydug pit
available for B tofill?
(a)50 (b)25 (c)20 (d)40
Q224. If 10 men and 4 boys working together can do twice as much work per hour as a man
and 4 boys together, then the ratio of the work done by a man to that of a boy for a
given time is
(a)2:1 (b) 1:2 (c) 3:4 (d) None ofthese
Q225. A is 50% more efficient than B and both are able to complete a work together in 15
days. How long would A, working alone, take to complete the work?
(a)20days (b)22days (c)24days (d) None ofthese
Q227. 16 men and 12 boys can complete a work in 26 days. 13 men can complete the same
work in 48 days. In how many days will 12 men and 6 boys complete half the work?
(a)30 (b)34 (c)39 (d)19.5
Q228. Two pipes can fill an empty tub in 5 minutes and 7 minutes respectively. An outlet
pipe can empty full tub in 3 minutes. If the tub is empty and all the pipes are opened
simultaneously when the tub will be full?
(a)11/4 hours (b)11/2 hours(c)2hours(d)13/4 hours
Q229. A cistern has a leak which would empty it in 3 hours. A tap is turned on which lets in
6litresofwaterperminute.Thetankisinitiallyfullwhenbothtapsareturnedonand
itisemptiedin6hours.Whatisthecapacityofthetank?
(a)2100litres (b)2160litres (c)2500litres (d) None ofthese
Q230. Two pipes can fill a tank in 4 hours and 8 hours respectively whereas a third pipe can
empty it in 6 hours .If the tank is initially empty and the three pipes are opened at
10:00 am, 11:00 am and 12:00 noon respectively, when will the tank be full?
(a)2:00P.M (b)1:30P.M (c)1:48P.M (d)None ofthese
TIME AND DISTANCE (AVERAGE SPEED, TRAIN, BOAT STREAM, RACES AND CIRCULAR TRACK)
Q232. A motorcyclist covered two-thirds of a total Journey at his usual speed. He covered
the remaining distance at three-fourth of his usual speed. As result, he arrived 30
minutes later than the time he would have taken at usual speed. If the total journey
was 180 km, then what was his usual speed?
(a)40kmph (b)36kmph (c)30kmph (d)32kmph
Q233. A man covers the distance between A and B in 30 minutes and returns from B to A
in 60 minutes the distance between A and B is 40 km, then what is average speed?
1 2
(a)26kmph (b)53 kmph (c)66 kmph (d) 70kmph
3 3
Q234. A person in a car moving at a speed of 64 km/h covers a certain distance in 9 hours.
How much time does another person travelling by a van at a speed of 72 km/h take
to cover the samedistance?
(a)6hours (b)12hours (c)8hours (d)7hours
Q236. A bus covers four successive distances of 45 km each at speeds of 10 kmph, 20 kmph,
30 kmph and 60 kmph respectively. What is the average speed of the bus?
Q237. An escalator is moving up at the rate of 12 ft/sec. If a man runs on it to reach the top
at the rate of 9 ft/sec. Given that when the escalator is not moving the man hasto
Q239. It takes 7 hours to cover a certain distance at a speed of 75 kmph by a car. At what
speed must one travel to cover the same distance in 2 hours less time?
(a)105kmph (b)534kmph (c)90kmph (d) 84 1kmph
7 7
Q240. In a journey, one –third of the distance is covered at 30 kmph and the remaining at
40 kmph. What is the average speed for the entire journey?
(a)35kmph (b)37.5kmph (c)36kmph (d)33.5kmph
Q241. A bus travels without any stoppages, form Ujjain to Indore. With an average speed of
60kmphwhilewithstoppagesitsaveragespeedforthejourneyisreducedto55kmph. Of
many minutes does the bus stop on an average perhour?
(a)20minutes (b)25 minutes (c)11minutes (d) none ofthese
Q242. A train moves a certain distance in 8 hours. If it had moved 5 kmph faster, it would
have covered the distance in 6 hours 40 minutes. What is the original speed of the
train?
(a)25kmph (b)40kmph (c)45kmph (d) 35kmph
Q243. The speed of a train is 15 m/s. The time it takes to cross a telephone pole is 10
seconds. What is the length of the train?
(a)100m (b)1.5km (c)150m (d)200m
Q244. A train 300 m long and moving at a speed of 25 m/s can cross a man standing on a
platform in
Q245. Two trains start at the same time from two stations A and B towards each other. They
arrive at B and A respectively in 5 hours and 20 hours after they passed each other. If
the speed of the train that started from A is 56 kmph, then what is the speed of the
second train?
Q246. A train leaves Hyderabad for Bangalore at 6:00 a.m. at a speed of 72 kmph. Another
trainleavesBangaloreforHyderabadat8:00a.m.ataspeedof54kmph.Howfarfrom
Hyderabad will the two trainsmeet?
(a) 144km
(b) 216km
(c) 314km
(d) Cannot bedetermined
Q247. Two trains are travelling in the same direction at 40 kmph and 60 kmph. The faster
train overtakes the slower train in 3 minutes. If the length of the faster train is 400 m,
then what is length of the slower train?
(a)200m (b)100m (c)150m (d) 600m
Q248. A train is 300 m long and moves at a speed of 50 km per hour. In what time does it
cross a person walking in the opposite direction at 4 km per hour?
(a)18sec (b)24sec (c)20sec (d) 15sec
Q249. A train which is 225 m long travels at 45 km/h. In what time does it cross a man
running at 18 km /hin the same direction?
Q250. Amanstandsonarailwaybridgewhichis300mlonghefindsthatatraincrosseshim
10 seconds and the bridge in 25 seconds. What is the length of the train ,
Q251. Two trains started from a station, at the same time in the same direction, at speeds
of 72 kmph and 60 kmph. What will be the distance between the two trains after 7
hours?
(a)42km (b)50.4km (c)63km (d)84km
Q252. In what time do two trains of lengths 380 m and 470 m, travelling in opposite
directions at respective speeds 55 km/h, and 35 km/h, cross each other?
Q253. Two trains are running on parallel lines in the same direction at a speed of 50 kmph
and 30 kmph respectively. The faster train crosses a man in the slower train in 18
seconds. The length of the faster train is train is.
(a)170m (b)100m (c)98m (d)85m
Q254. A man can row at 12 kmph in still water. He finds that it takes him thrice as much
time to row up the river as it takes to row down the river. What is the speed of the
current?
(a)6kmph (b)24kmph (c)48kmph (d)18kmph
Q255. A man can row a certain distance against the stream in six hours. However, he would
take two hours less to cover the same distance with the current. If the speed of the
current is 2 kmph, then what is the speed of the man in still water?
(a)10kmph (b)12kmph (c)14kmph (d) 8kmph
Q256. A man can row three quarters of a kilometer against the current in 11 ¼ minutes
and return in 7 ½ minutes. Find the speed of the man in still water.
(a)8kmph (b)7kmph (c)6kmph, (d) 5kmph
Q257. A man can row downstream at 12 kmph and upstream at 8 kmph. Find the ratio of
the speed of the current to the speed of the man in still water.
Q258. Apersoncanrowat9kmperhourinstillwater.Hefindsthatittakeshimtwiceas
muchtimetorowupstreamascomparedtodownstream.Whatisthespeedofthe current?
(a)3kmph (b) 4½ kmph (c)6kmph (d) 3 ½ kmph
Q259. Amancanrowat6kmphinstillwaterandariverisflowingat4kmph.Howlong
willthemantaketogotoaplace1kmdownstreamandreturn?
(a)36minutes (b)24minutes (c)12minutes (d) 18minutes
Q260. A person can row a distance of one km against the - stream in ten minutes and
along the stream in four minutes. What is the speed of the stream?
(a)3km/h (b)9km/h (c)5.6km/h (d)4.5km/h
Q261. Apersoncanrow18kmdownstreamand27kmupstreaminthreehoursand18
kmupstreamand27kmdownstreamintwohours42minutes.Whatisthetime
takenbythepersontorow25kmupstream?
(a) 17hours (b) 11hours (c)2hours (d)2 3hours
10 2 10
Q262. It takes eight hours for a 600 km journey, if 120 km is done by train and the rest by
car.Ittakes20minutesmore,if200kmisdonebytrainandtherestbycar.Theratio
ofthespeedofthetraintothatofthecarsis:
(a) 3: 4 (b) 3: 2 (c) 2:3 (d) 4 :3
Q263. Apersoncanrowataspeedof18kmphinstillwater.Forthesamedistance,thetime taken
by him to row upstream is thrice the time taken by him to row downstream.
Whatisthespeedofthestream?
(a)9km/h (b)6km/h (c)8½km/h (d)3km/h
Q264. Speed of sound is 330 m/s in air while it is 440 m/s in a liquid. the total time
Q266. InaKmraceAbeatsBby50m.Ina3kmraceAbeatsBby
(a)150m (b)300m (c)400m (d)600m
Q267. InAkilometerrace,ifAgivesBastartof100meters,bothofthemreachthefinish
lineatthesametime,ifA`sspeedis20m/s,themwhatisB`sspeed?
(a)16m/s (b)17m/s (c)18m/s (d)19m/s
Q268. Inakilometerrace.AcangiveBastartof40mandBcangiveCastartof25m(In
thesamerace)howmanymetresstartcanAgiveC.
(a)46m (b)56m (c)66m (d)64m
Q269. Inakilometerrace,AbeatsBby50metresandBbeatsCby75metres.Byhowmany
metresdoesAbeatCinthesamerace?
Q271. After how much time from the start all three will meet again at the starting point?
(a)24seconds (b)30seconds (c) 60 seconds (d) 120 seconds
Answer:D
Solution: For meeting at the starting point:
TimetakentomeetforthefirsttimebyAandB= 𝐿𝑒𝑛𝑔𝑡𝑜𝑓𝑡𝑒𝑡𝑟𝑎𝑐𝑘
𝑅𝑒𝑙𝑎𝑡𝑖𝑣𝑒 𝑠𝑝𝑒𝑒𝑑 𝑜𝑓 𝐴 𝑎𝑛𝑑 𝐵
= 120/5 = 24 seconds.
Answer: B
Solution:
Answer: A
Solution:
Answer: B
Solution:
Q277. P, Q, R run around a circular track 1200 m long with speed of 9, 18, 27 kmph.If they
Answer: B
Solution: L = 1200 m
Speed of P(p) = 9 x 5/18 = 2.5 m/sec
Whatistheperimeterof∆ABCincm?
Answer: 29
Q279. In the given figure ABCD is a Parallelogram then the find out of the value of x is?
Q280. In the given Figure ∠PQA = 20° and ∠APQ = 120° then what is the value of
∠PAQ?
Answer: 40°
Q281. For a triangle ABC, D and E are two points on AB and AC such that AD = 1/4 AB,
AE =1/4 AC. If BC =12 CM, then DE is
(a)5cm (b)4cm (c)3cm (d)6cm
Answer:C
Solution:
Q283. In triangle ABC, a line is drawn from the vertex A to a point D on BC. If BC =9 cm
and DC = 3 cm, then what is the ratio of the areas of triangle ABD and triangle ADC
respectively?
(a)1:1 (b)2:1 (c)3:1 (d)4:1
Answer: B
Solution:
Q285. What is the length of the inradius of a triangle of sides 3 cm, 4cm and 5 cm?
(a)2cm (b)2.5cm (c)1.8cm (d) 1 cm
Answer:D
Solution: Area = r * (a + b + c)/2
6 = r (6).
So, r = 1 cm.
Inthefigureabove,ABCisanequilateraltriangle.Thebisectorsof∠ABCand∠ACB
meetatD.WhatisthevalueofBDCindegree?
Answer: 120
Q288. Averticalstick20cmlongcastsashadow15cmontheground.Whatisthelengthof
alamppost,ifitcastsashadow60cmlongtheground?
(a)20cm (b)40cm (c)60cm (d) 80cm
Answer: D
Solution:
Q290.
In the above figure 𝑙1 and 𝑙2 are parallel lines. ∠𝐴𝐵𝐶= 80°. If the lines BD and CF are parallel
and. ∠𝐷𝐵𝐶 = 30°, then what is ∠𝐹𝐶𝐸?
Q291.
In the above figure, l1 and l2 are parallel lines. What is the value of y?
(a)15° (b)30° (c)45° (d)60°
Q293. In the above figure, the point C divides AE and BD in the ratio 2:1. What is ∠𝐴𝐶𝐵
given ∠𝐶𝐷𝐸 = 20°, ∠𝐵𝐴𝐶=40°?
Q294.
IntheabovefigurethelinePSbisects∠𝑄𝑃𝑅,𝑎𝑛𝑑∠𝑃𝑆𝑄=∠𝑃𝑆𝑅,and
Q296.
Intheabovefigure,∠𝑄𝑃𝑆=2∠𝑆𝑃𝑅=∠𝑃𝑅𝑆,Find∠𝑄𝑃𝑅.
(a)70° (b)80° (c)90° (d)100°
Q297. The sum of the interior angles of a regular polygon is 24 times the exterior angle.
What is the number of sides of the polygon?
(a)4 (b)6 (c)8 (d)12
Q298. Ifeachinteriorangleofaregularpolygonis150°,thenwhatarethenumberdiagonals?
(a)54 (b)48 (c)66 (d)60
Q299. In a triangle ABC, the in center is O. If ∠𝐵𝐶𝐸 =100°, ∠𝐵𝐴𝐶 is equal to
Intheabovefigure,if2∠𝑃𝑂𝑅=3∠𝑅𝑂𝑄and∠𝑃𝑂𝑄=100°.Thenwhatis∠𝑃𝑂𝑅? (a)30°
Intheabovefigure,QPandQRaretangentstothecircle.If∠𝑃𝑄𝑅=50°,thenwhatis
∠𝑃𝑂𝑅?
(a)130° (b)120° (c)110° (d)100°
Q302.
In the above figure, AB is a tangent to the circle O. If CD bisects ∠𝐴𝐶𝑂 then what is
∠𝐶𝑂𝐷?
Q304.
Intheabovefigure,Oisthecenterofthecircle.∠OQP+∠ORP=70. What is
theORQ?
(a)20° (b)30° (c)40° (d)50°
Q305.
In the above figure, AB is the diameter of the circle AC and BD are tangents drawn to
thecircleatAandB.Whatisthe∠𝐴𝐶𝐷,if∠CDB=70°?
Q307.
Inthefigureabovel1andl2areparallellines.Thebisectorsof∠DACand∠ACE
MeetatB.IfAB=40cmandBC=9cm,thenwhatisthelengthofAC?
Q308.
In the above figure, ∠PSR = ∠PQT, ∠TPQ =60°, ∠PQT = ∠PRS + 40°. What is ∠PRS?
(a)30° (b)40° (c)80° (d) 60°
Q309.
Inthefigureabove,CFandDGarethebisectorsof∠BCDand∠BDCrespectively. What is
the value of angle∠GFC?
Q311.
In the above figure, O is the center of the circle, AB is a tangent to the circle and
Q313. The ratio of the length, breadth and height of cuboid is 5 : 4 : 2. What is the lateral
surface area (in m2) of the cuboid if the length of the longest diagonal that fits in the
cuboid is 15√5m?
Answer: 900
Q314. The ratio of the lengths of two cuboids is 30 : 11, that of their breadths is 11: 13, and
that of their heights is 26 : 15. What is the ratio of their volumes?
(a)1 : 4 (b) 1: 2 (c) 2:1 (d) 4 : 1
Q315. The length of the longest rod which can fit into a cubical room of volume 2197 m 3, is
Answer:6
Q317. A solid metallic spherical ball of diameter 6 cm is melted and recast into a cone with
diameter of the base as 12 cm. The height of the cone is .
Q318. If the volume of a sphere is divided by its surface area, the result is 27 cm. The radius of
the sphere is:
(a)9cm (b) 36cm (c)54cm (d) 81cm
Q319. A right circular cone and a right circular cylinder have equal base and equal height. If the
radius of the base and the height are intheratio 5:12, then the
ratio of the total surface area of the cylinder to that of the coneis
(a)29:18 (b)13:9 (c)17:9 (d) 34:9
Q320. A hollow spherical metallic ball has an external diameter 6 cm and is 1cm thick.
2
Q321. The number of coins of radius 0 .75 cm and thickness 0.2 cm to be melted to make right
circular cylinder of height 8 cm and base radius 3 cm is:
Q326. The radius of the cylinder is half its height and surface area of the cylinder is616 sq. cms.
What is volume of the cylinder incm3?
Answer: 2156
Q327. The distance between the centers of two circles touching each other internally is 7 cm.
What is the area common to both the circles, if ratio of their radii is2 : 1? (in cm 2)
(a)154 (b)144 (c)6164 (d)1386
Answer: A
Q329. What is the area of the shaded region, if O is the center of the circle and OA = 12
cm and AOB = 120?
𝑐𝑚2
Q331. At what time between 7 and 8 o'clock will the hands of a clock be in the same
straight line but, not together?
(a) 5 min.past
2
(b) 5 minutes past7
11
3
(c) 5 minutes past7
11
5
(d) 5 minutes past7
11
Answer: D
Q332. At what time between 2 and 3 o'clock will the hands of a clock be together?
109
(a) min past2
11
120
(b) min past2
11
131
(c) min past2
11
142
(d) min past2
11
Answer: B
Q333. An accurate clock shows 7 a.m. Through how many degrees will the hour hand
rotate when the clock shows 1 p.m.?
(a)154° (b)180° (c)170° (d)160°
Answer: B
Solution:Weknowthatangletracedbyhourhandin12hrs.=360° From 7
to 1, there are 6hours.
Angle traced by the hour hand in 6 hours =6*(360/12)=180°
Option B is the correct answer.
Q335. What is the time between 3 and 4 will the hands of a watch point in the opposite
direction?
(a) 49(1/11) min past3
(b) 49(3/11) min past3
(c) 49(2/11) min past3
(d) 49(4/11) min past
3Answer:A
Q336. A clock is started at noon. By 10 minutes past 5, the hour hand has turned through:
(a)145° (b)150° (c)155° (d)160°
Answer: C
Solution:
Q337. How many times are the hands of a clock at right angle ina day?
Answer:44
Q338. How many times do the hands of a clock coincide inaday?
Answer:22
Q341. If today is a Monday then what will be the day after 68 days?
(a)Tuesday (b)Wednesday (c)Thursday (d)Saturday
Q342. [MSQ]
Which of the following years are leap years?
(a)700 (b)800 (c) 2000 (d)2100
Answer: b, c
Q343. 17th June, 1998 wasa
(a)Monday (b)Tuesday (c)Wednesday (d)Thursday
Q344. If it was Thursday on Aug 15, 2012, then what was the day on June 11, 2013?
(a)Wednesday (b)Monday (c)Saturday (d)Tuesday
Q345. What was the day of the week on 7th September, 1990?
(a)Wednesday (b)Thursday (c)Friday (d)Saturday
Q348. If6thMarch,2005isMonday,whatwasthedayoftheweekon6thMarch,2004?
(a)Sunday (b)Saturday (c)Tuesday (d)Wednesday
Q350. If the seventh day of a month is three days earlier than Friday, What day will it be on
the nineteenth day of the month?
(a) Sunday
(b) Tuesday
(c)Wednesday
(d)Monday
Answer: A
Solution:
The seventh day of the month is three days earlier than Friday, which is Tuesday.
So, the fourteenth day is also Tuesday and thus, the nineteenth day is Sunday.
Q351. XwasbornonMarch6,1993.ThesameyearIndependenceDaywascelebratedon Friday.
On which day was Xborn?
(a)Monday (b)Wednesday (c)Thursday (d)Friday
Answer: C
Solution:
NumberofdaysfromMarch6,1993toAugust15,1993. Mar
Apr May June JulyAugust
= 25 + 30 + 31 + 30 + 31 +15
= 162 days = 23 weeks + 1 day.
Clearly, the day on March 6 will be the same as on August 14, i.e., Thursday.
Q354. The calendar of the year 2024 can be used again in the year?
(a)2030 (b)2052 (c) 2048 (d)2036
QUADRATIC EQUATION
Q356. The roots of the equation 3x2 - 12x + 10 = 0 are?
(a) rationalandunequal (b)real andequal
(c) irrationalandunequal (d) rational andequal
Q357. If the roots of a quadratic equation are 20 and -7, then find the equation?
(a) x2 + 13x - 140 = 0
(b)x2 - 13x + 140 = 0
(c) x2 - 13x - 140 = 0
(d) x2 + 13x + 140 = 0
Answer: C
Solution:
Any quadratic equation is of the form
x2-(sumoftheroots)x+(productoftheroots)=0 ------------ (1)
wherexisarealvariable.Assumoftherootsis13andproductoftherootsis-140,
thequadraticequationwithrootsas20and-7is:x2-13x-140=0.
Q358. If the sum and the product of the roots of the quadratic equation x 2 + 20x + 3 = 0 are
𝛼and𝛽,thenwhatthevalueof(𝛼+𝛽)−𝛼𝛽?
Answer: 43
Q359. The sum of the squares of two consecutive positive integers exceeds their product by
91. Find the integers?
(a)9,10 (b)10,11 (c) 11,12 (d) 12,13
Answer: A
Solution: Let the two consecutive positive integers be x and x + 1
x2 + (x + 1)2 - x(x + 1) = 91
x2 + x - 90 = 0
(x + 10)(x - 9) = 0 => x = -10 or 9.
As x is positive x = 9
Hence the two consecutive positive integers are 9 and 10.
GENERAL APTITUDE PAGE 83
Q360. Two equations (I) and (II) are given. You have to solve both the equations and find
relation between x and y.
I. x2 – 0.5x – 39 = 0
II. y2 – 15.5y + 60 = 0
(a)𝑥=𝑦 (b)𝑥<𝑦 (c)𝑥>𝑦 (d) can‟t say
Solution:
I. x2 – 0.5x – 39 = 0
x2 + 6x – 6.5x – 39 = 0
x(x + 6) – 6.5(x + 6) = 0
(x – 6.5)(x + 6) = 0
x = 6.5, – 6
II. y2 – 15.5y + 60 = 0 y2
– 8y – 7.5y + 60 = 0 y(y
– 8) – 7.5(y – 8) =0
(y – 8)(y – 7.5) = 0
y = 7.5, 8
Therefore, for any value of x and any value of y
x < y.
Q361. Two equations (I) and (II) are given. You have to solve both the equations and find
relation between x and y.
I. x3 – 14 – 1714 =0
II. 3y2 – 63 – 300 =0
(a)𝑥=𝑦 (b) 𝑥<𝑦 (c)𝑥 >𝑦 (d) can‟tsay
Q363. If the speed of a car is increased by 15 kmph, it takes one hour less to cover a
distance of 300 km. What is the original speed of the car?
(a)40kmph (b)50kmph (c)60kmph (d) 75kmph
Q364. What are the value of x that satisfy the inequality 3x2 + 8x + 5 >0?
(a)5> 𝑥 > −1
3
(b)−5<𝑥<−1
3
(c)−5>𝑥<−1
3
5
(d)− > 𝑥 and 𝑥 > −1
3
Q365. The two sides of a right –angled triangle containing the right angle are 3x+4 and
2x+3. If the area of the triangle is 20 m2 then what is the hypotenuse of the triangle?
Q366. Whatissumoftherootsoftheequation10x4-63x3+52x2+63x+10=0?
Answer:6.3
Q367. A person on a tour has Rs 9,600 for his expenses. If his tour is extended by sixteen
days, he has to cut done his daily expenses by Rs 20. What is the original durationof
the tour indays?
Answer: 80
Q368. If one root of the quadratic equation x2 + 11x + k = 0 is 4, find the other root and the
value of k (respectively).
11
Q369. If x < 0, what the maximum value of 𝑥 + ?
11 𝑥
𝑥 𝑥
Q371. The roots of the equation (2 + √3) +(2 − √3) = 14 are
(a)2,-1
(b)1,2
(c)2,-2
(d) none of these
2 2
Q372. If P is rational number and 12 (𝑃 + 1) = 120 + 5 (𝑃 − 1) , then what is P?
𝑃 𝑃
Q373. What is the sum of all possible value of x, for which the following equation satisfies?
32𝑥−12.3𝑥+27=0
Answer: 3
(a) 3,-3/9
(b) 6,-2/9
(c) 8, 2/5
(d) 16, 1/5
1 10
Q377. If√2𝑥−3+ = then 𝑥 is equalto
√2𝑥−3 3
LOGARITHMS
Q381. [MSQ]
If 𝑥=1+log 𝑎𝑏𝑐,𝑦=1+log 𝑏𝑐𝑎,𝑧=1+log 𝑐𝑎𝑏,thenwhichofthefollowing
statement is are true?
(a) 𝑥𝑦𝑧 + 𝑥𝑦 + 𝑦𝑧 + 𝑧𝑥 = 0
(b) 𝑥𝑦 + 𝑦𝑧 + 𝑧𝑥 − 𝑥𝑦𝑧 = 0
1 1
(c) 1 + + = 0
𝑥 𝑦 𝑧
1 1
(d) 1+ + = 1
𝑥 𝑦 𝑧
Answer: b, d
Answer: 1
Q383. Forwhichvalueofxtheequation:3𝑥log52+2𝑙𝑜𝑔5𝑥=64,satisfies?
Answer:625
Q384. The number of the solutions of log4(𝑥 − 1) = log2(𝑥 − 3) is
(a)3 (b)1 (b)2 (d)0
Answer:10
PROGRESSIONS (SERIES)
Q386. The interior angles of a polygon are in arithmetic progression. The smallest angleis
120 and the common difference is 5. What is the number of sides of the polygon?
(a)16 (b)9 (c)7 (d) Both a &b
Answer: B
Q387. IfthesumofthreenumbersinH.Pis37andthesumoftheirreciprocals0.25,then the
product of these numbers willbe
Answer: 1800
Answer: 131
Answer: 4960
Q391. Consider three terms in arithmetic progression such that their sum is 33 and product
is1155.Whatiscommondifference?
Answer: 4
Q392. The sum to n terms of a series in AP is 6𝑛2+ 6𝑛. What is the 4thterm of the series?
Answer: 48
Q393. Three terms are in geometric series such that their sum is 49 and product 2744.What
is the sum of first term and commonratio?
Answer: 9
(𝑐+𝑏)
Q395. If (𝑎+𝑏),𝑏, areinAP,thena,1,carein
1−𝑎𝑏 1−𝑐𝑏 𝑏
value of x?
(a)0.2 (b) 19/7 (c) Both a&b (d)can‟t
determined
Q398. If 𝑆∞= 1 + 22x + 32x2 + 42x3 + …….∞; where |x|<1, then 𝑆∞ is equivalent to
Answer:131054
Q400. Whatis60thtermoftheseries1+2+5+12+25+46+…?
Answer: 68500
Q402. [MSQ]
Howmanytermsseries54+51+48+45+…mustbetakentomake513? (a)16
(b)17 (c)18 (d)19
Answer: c, d
Q404. Whatisthevarianceof1,2,3,4,5?
Answer: 2
Q405. Whatisthestandarddeviationof1,2,3,4,5,6?
Answer: 1.71
Q406. What is the range and quartile deviation from the following marks of 13 students in
Mathematics?
60, 40, 10, 72, 85, 96, 38, 41, 52, 28, 9, 35, 25.
Q408. What is the mean deviation about the mean for the following data:
12, 18, 25, 30, 35
(a)3.5 (b)7.2 (c)7.5 (d)6.8
Answer: B
Solution: Mean = 1/5 (12 + 18 + 25 + 30 + 35) = 120/5 = 24
Mean Deviation = 1{|12 – 24|+|18 – 24|+|25 – 24|+|30 – 24|+ |35 – 24|}
5
= 36/5 = 7.2
Q410. The variance of the numbers -35, -30, -5, -20, -15 is
(a)20 (b)50 (c)25 (d)114
Q411. If the sum of the squares of 8 observations is 144 and their mean is 3, then the
standard deviation is
(a)9 (b)3 (c)1.73 (d)81
Q412. How many integers satisfying the inequality
(𝑥2−14𝑥+45)(𝑥2−4𝑥−21)(𝑥2−11𝑥+10)<0?
Answer:4
Q413. Whatistherangeofallrealvalueofxif||x–3|+5|<4?
(a)(3,12)
(b) (-6,4)
(c) (0, 5)
Q416. If (𝑥−3)>5,thenthesolutionsetofxfortheinequalityis
(𝑥+2)
(a)(2, 13/4)
(b)(-2,13/4)
(c) (-13/4,2)
(d) (-13/4, -2)
DATA INTERPRETATION (TABLE, PIE CHART, BAR DIAGRAM AND LINE GRAPH)
Directionfornextfourquestions:Studythefollowingtablechartcarefullyandanswerthe
questions given beside.
The table given below shows the percentage of appeared and qualified candidates in a
competitive examination from different institutes.
52 × 4
Reqd. ratio = = 13 : 75
50 × 24
Hence, option D is correct.
Q418. What percentage of the candidates from institute C has been declared qualified out
of the total candidates appeared from this institute?
(a)30% (b)25% (c)40% (d)20%
Answer: D
Solution:
Qualified candidates from institute C = 18% × 4000
Appeared candidates from institute C = 15% × 24000
18 × 4 × 100
Reqd. % = = 20%
15 × 24
Hence, option D is correct.
Q419. What is the percentage of students who qualified from the institute C and D together
with respect to those who appeared from the institute C and D together?
(a)24.98% (b)30.98% (c)20.98% (d)31.98%
Answer: C
Solution:
Qualified candidates from C and D together = {18 + 16} % × 4000
Appeared candidates from C and D together = {15 + 12} % × 24000
34 × 4 × 100
Reqd. % = = 20.98%
27 × 24
Hence, option C is correct.
18 × 4
A=
24 × 25
12 × 4
B=
10 × 24
18 × 4
C=
15 × 24
16 × 4
D=
12 × 24
20 × 4
E=
18 × 24
16 × 4
F=
20 × 24
In these values 4/24 can be discarded because it is common in comparison.
So, here, A and F are less than 1, so they are removed without solving.
B = 1.2, C = 1.2, D = 1.33, E = 1.1
D has the highest percentage of qualified candidates with respect to those appeared.
Hence, option B is correct.
Directions for next four questions: Following table shows the mobile addicted population
ofsixdifferentcitiesMumbai,Delhi,Jaipur,Kochi,PuneandAgraandtheratioofMalesto Females
among them and the ratio of Adults to Children in the total mobileaddicted
population. Answer the given questions based on this table
Q422. Whatisthedifferencebetweenthetotalnumberofadultandthetotalnumber
childrenmobileaddictedpopulationofMumbai?
(a)0.918lakh (b) 1.026lakh
(c)1.124lakh (d) 1.107lakh
Answer: B
Solution: ∴ Required Difference
(5 – 3)
= 410400 ×
8
= 1.026 lakh
Hence, option B is correct.
Q423. Which of the following cities has the maximum addicted adult population?
(a) Pune
(b) Kochi
(c) Jaipur
(d) Delhi
Answer:D
Solution:AdultpopulationofMumbai-2.565lakh,Delhi-2.877lakh,
Jaipur-2.583lakh,Kochi-2.66lakh,Pune-2.775lakh
Hence, Delhi has the maximum number of mobile addicted adults.
Hence, option D is correct.
Q427. How many people, in this group, have blood types A orB?
Answer: 82
Solution: (16% + 25%) × 200 = 41 × 200 /100 = 82
Q428. Number of students appearing from B state in 2016 was, what percent of that from
state F in 2015?
(a)94 (b)95 (c)96 (d)97.
Answer: C
Solution:
Number of students appearing from B state in 2016 = 20*240000/100 = 48000
Number of students appearing from F state in 2015 = 20*250000/100 = 50000
Percentage = 48000*100/50000 = 96%
Q429. The total number of students appearing from B & C together in 2015 is
approximately equal to that from which of the following pairs of states in 2016?
(a) A&F (b) C&D (c) A&C (d) D &F
Answer: C
Solution:
StudentsappearingfromBandCin2015=(18+12)*250000/100=75000 Go
through options verification check one byone
You will find option C is approximately equal to 75000
Q430. What is total number of males working in department Manager, HR and Clerk?
(a)400 (b)380 (c)394 (d)396
Q431. What is the ratio of females working in department Manager and clerk and males in
department HR and marketing?
(a)56:37 (b)56:35 (c)55:37 (d)55:30
Q434. What is the ratio between total number of female employees working in all the
department together and males working in all the department?
Data for next five questions: Study the graph carefully and answer the following question.
Data related to Income (in Rs thousands) and Expenditure (in Rs thousands) of company
during six months.
Q436. What is total loss percentage incurred by company in March and April?
(a)40 (b)50 (c)30 (d)45
Q437. Profit earned by company in Feb is by what percent more than profit earned by
company in May.
(a)443 (b)488 (c)384 (d)444
9 9 9 9
Q439. If company income increased by 20% from June to July and Expenditure decrease by
10% .What was his profit percent in month of July?
(a)4.76 (b)2.56 (c)5.88 (d)3.17
Q440. For which of the following pairs of years the total exports from the three Companies
together are equal?
(a) 1995and1998 (b) 1996 and1998
(c) 1997and1998 (d) 1995 and1996
Answer: D
Solution:
Q442. In which year was the difference between the exports from Companies X and Y the
minimum?
(a)1994 (b)1995 (c)1996 (d)1997
Answer: C
Solution:
Q444. In how many of the given years, were the exports from Company Z more than the
average annual exports over the givenyears?
Answer: 4
Solution:
Average annual exports of Company Z during the given period
1
= x (60 + 90 + 120 + 90 + 60 + 80 + 100)
7
600
=Rs. crores
7
= Rs. 85.71crores.
Q448. What is the average sales of all the branches (in thousand numbers) for the year
2000?
(a)73 (b)80 (c) 83 (d)88
Answer: B
Solution:
Average sales of all the six branches (in thousand numbers) for the year 2000
1
= x [80 + 75 + 95 + 85 + 75 + 70]
6
= 80.
Q450. What is the total number of male employees taking all the banks together?
(a)4060 (b)4120 (c)4180 (d)4280
Answer: C
Solution:
Q452. Approximately by what percent is the number of male employees working in banks
A and C together more than that of the total number of female employees working in
bank B andD?
(a)9% (b)15% (c)11% (d)13%
Answer: D
Solution:
Q454. Approximately by what per cent is the number of total employees in bank C more
than that of bank D?
(a)8% (b)6% (c)4.4% (d)10%
Answer: B
Solution:
Q455. The following line graph gives the percentage of the number of candidates who
qualified an examination out of the total number of candidates who appeared for the
examination over a period of seven years from 1994 to 2000.
Q457. A student was asked to multiply a given number by 8/17 . Instead, he divided the
given number by 8/17 . His answer was 225 more than the correct answer. The given
number was
(a)64 (b)289 (c)136 (d)225
Q458. On dividing 2272 as well as 875 by 3-digit number N, we get the same remainder.
The sum of the digits ofNis _
(a)13 (b)12 (c)11 (d)10
Answer: D
Solution:
(2272-875) = 1397, is exactly divisible by N.
Now , 1397 = 11 x 127
The required 3-digit number is 127,the sum of digits is 10.
Q461. It is being that (232+ 1 )is completely divisible by whole number. Which of the
following numbers is completely divisible by this number?
(a)(216+1) (b)(216−1) (c)(7×233) (d) 296 +1
Answer: D
Solution:Let𝑥=232 ,𝑡𝑒𝑛(232+1)=𝑥+1
296 + 1 = (232)3+ 1 = (𝑥3 + 1) = (𝑥 + 1)(1 + 𝑥 + 𝑥2);
This is divisible by x +1. Therefore, (296+ 1 )is completely divisible by (232+ 1 ) .
Answer: 9
Q464. [MSQ]
LetN=(n–2)(n–1)n(n+1)(n+2);wherenisanaturalnumber.Whichofthe following
statement(s) is/aretrue?
(a) The unit digit of N is0.
(b) N is perfectly divisible by24.
(c) N is perfect square.
(d)(N+1)2mod10=1.
Answer: a, b, d
Data for next two questions: A trader gives a discount of 16% and earns a profit of 40%.If
the difference between the marked price and the cost price is Rs 400, then
Q467. Whatwillbethesellingprice(inRs)?
Answer: 840
100
Solution: Let MP = Rs 100, then CP = 84 × = 𝑅𝑠 60
140
MP – CP = 100 – 60 = 40
Since, the difference between the marked price and the cost price is Rs 400, hence
40% of MP=400 => 𝑀𝑃 = 𝑅𝑠1000
Therefore SP = 1000 × 0.84 = Rs 840.
Q469. A shopkeeper marked a product, 20% above the cost price and sold the product for
Rs. 3888 by giving two successive discounts of 10% each. Find the cost price of the
product and the loss percentage of the shopkeeper.
(a)4200,7.4% (b)4000,2.8% (c)4100,5.1% (d) 4050,4%
Answer: B
Solution:
Q470. The marked price of a John Players Denim is Rs. 1490. On the occasion of
Janmashtami, the showroom offered two successive discounts of 11.25% and D%
respectively. If an individual purchased the Denim for Rs. 1193.44, then find the
value of value of „D‟?
(a)9.75 (b)8.75 (c)5.5 (d)10.75
Answer: A
Solution:
Answer: 11780
Solution:
Answer: 2340
Solution:
Q474. There four articles are sold at the same price (i.e., the selling prices are equal).
Firstoneissoldataprofitof20%,
Secondoneissoldataprofitof20%,
Thirdoneissoldatalossof20%,and
Fourth is sold at a loss of20%.
What is the overall percentage of profit/loss?
(a)0%loss (b) 4%profit (c)4%loss (d) Insufficient
dataAnswer:C
Solution:
1 1 1 1
(+ − − )
Required %ofloss==1.21.20.80.8×0.2×100=4%
1 1 1 1
1.2+1.20.+80.8+
Q476. Rs.39030isdividedbetween„a‟and„b‟insuchawaythatamountgivento„a‟onC.I.
in7yearsisequaltotheamountgivento„b‟onC.I.in9years.Findthepartof„a‟.If the rate of
interest is4%.
(a)20200 (b)20900 (c)20280 (d)20100
Answer: C
Solution:
Q477. What will be the difference between simple and compound interest on a sum of Rs.
4500 put for 2 years at 5% per annum?
(a)12.45 (b)12.95 (c)11.25 (d)10.25
Answer: C
Solution:
Q479. The compound interest on Rs. 10,000 in 2 years at 4% per annum the interest being
compounded half-yearly, is :
(a)824.3216Rs (b)804.3216Rs (c)814.3216 Rs (d) 834.3216Rs
Answer: A
Solution:
Q480. Thecompoundinterestonacertainsumofmoneyinvestedfor2yearsat5%per
annumisRs.328.Thesimpleinterestonthesum,atthesamerateandforthesame period
willbe?
(a)340 (b)320 (c)330 (d)390
Q481. AsumofRs.2000amountstoRs.4000intwoyearsatcompoundinterest.Inhow
manyyearsdoesthesameamountbecomesRs.8000?
(a)2years (b)4years (c)6years (d) 8years
Answer: B
Solution:
Q482. A, B and C are partners. A got the (2/3)rd of the total profit. B and C got the rest
amount in equal ratio. When the rate of profit is increased from 5 to 7%, there is a
increase of Rs. 400 in profit of A. What will be the profit of B?
(a)Rs.500 (b)Rs.250 (c)Rs. 480 (d) Rs.350
Answer: D
Solution: Let the cost = 100x, i.e. Profit = 5% of 100x =5x
A's profit =(2/3)*5x
B'sprofit=(1/2)*(1/3)*5x=(1/6)*5x
C'sprofit=(1/2)*(1/3)*5x=(1/6)*5x
NewProfit=7%of100x=7x
i.e.NowA'sprofit=(2/3)*7x
ChangeinA'sprofit=(2/3)*7x-(2/3)*5x=(4/3)x
Given, (4/3)*x =Rs.400
Q483. P,QandRsignedforaprojectofRs.23600.PinvestedRs.5000morethanQandQ
investedRs.3000morethanR.IftheprofitisRs.35400,findtheshareofQ.
(a)Rs.21800 (b)Rs.18000 (c)Rs.10800 (d) Rs.11400
Q484. In a mixture of milk and water the proportion of water by weight was 75%. If in 60
gm of mixture 15 gm water was added, what would be the percentage of water?
(Weight in gm)
(a)80% (b)70% (c) 75% (d)62%
Answer: A
Solution:
Water in 60 gm mixture=60 x 75/100 = 45 gm. and Milk = 15 gm.
Afteradding15gm.ofwaterinmixture,totalwater=45+15=60gmandweightof
amixture=60+15=75gm.
So % of water = 100 x 60/75 = 80%.
Q485. The average weight of boys in a class is 30 kg and the average weight of girls in the
sameclassis20kg.Iftheaverageweightofthewholeclassis23.25kg,whatcouldbe
thepossiblestrengthofboysandgirlsrespectivelyinthesameclass?
(a) 14and16 (b) 13and 27 (c) 17and27 (d) None ofthese
Answer: B
Solution:
Q486. How many litres of pure water must be mixed with 175 litres of pure spirit in order
to gain (100/7) % by selling the spirit it at its cost price?
(a)24litres (b)25litres (c)50litres (d) 75litres
Answer: B
Q488. ThreecarsleaveAforBinequaltimeintervals.TheyreachBsimultaneouslyandthen
leaveforPointCwhichis240kmawayfromB.ThefirstcararrivesatCanhourafter the
second car. The third car, having reached C, immediately turns back and heads
towardsB.Thefirstandthethirdcarmeetapointthatis80kmawayfromC.Whatis
thedifferencebetweenthespeedofthefirstandthethirdcar?
(a)60kmph (b)20kmph (c)40kmph (d) 80kmph
Answer: A
Solution: Create a visual of where the three cars are and how they travel between the
points.
Q489. Ankit when going slower by 15 Km/hr, reaches late by 45 hours. If he goes faster by
10 Km/hr from his original speed, he reaches early by 20 hours than the originaltime.
What is the distance he covered byAnkit?
(a)8750Km (b)9750Km (c)1000Km (d) 3750Km
Q490. Three friends A, B and C decide to run around a circular track. They start at the same
time and run in the same direction. A is the quickest and when A finishes a lap, it is
seen that C is as much behind B as B is behind A. When A completes 3 laps, C is the
exact same position on the circular track as B was when A finished 1 lap. What is the
ratio of the speeds of A, B and C?
(a) 5 : 4:2 (b) 4 : 3:2 (c) 5 : 4:3 (d) 3 : 2 :1
Q491. A,B,C,andDaremovingonacirculartrackinthesamedirection.Allstartedrunning
simultaneously and from the same point such that the ratio of their speeds is 1 : 2 : 3:
4. At how many distinct points any two of themwillmeet?
Answer:4
Solution:
Therewouldbe4C2=6differentcombinationtakingtwoatatime.Let‟stakethemone byone.
A andB:
Thespeedsareinthesimplestformandnumberofdistinctpointstheymeet=2-1=1. A andC:
The speeds are in the simplest form and number of distinct points they meet = 3-1 = 2.
A and D:
Q492. Two persons A and B are racing along a circular track. The speed of A is twice the
speedofB.Thelengthofthecirculartrackis800mandthelengthoftheraceis7200
m. After the start of the race, A meets B for the first time at the end of the third
minute. If A and B start the race from the same point, What is the time taken by A to
finish the race?
(a)19minute (b)9.5minute (c)27minute (d)13.5minute
Answer:D
Solution:
Q494. A house has two wall clocks, one in kitchen and one more in the bedroom. The time
displayed on both the watches is 12.A.M right now. The clock in the bedroom gains
fiveminuteseveryhour,whereastheoneinthekitchenisslowerbyfiveminutesevery hour.
After how many hours the watches show the same timeagain?
(a)24hours (b)48hours (c)72hours (d) 96
hoursAnswer:C
Solution: The faster clock runs 5 minutes faster in 1 hr.
The slower clock runs 5 minutes slower in 1 hr.
Therefore, in 1 hour, the faster clock will trace 5+5=10 min more when compared to
the slower clock.
The table given below depicts the time difference between the slower and faster
clock:
Q497. The vlaue of 𝛼 for which one root of the quadratic equation
(𝛼2−5𝛼+3)𝑥2+(3𝛼−1)𝑥+2=0,istwiceaslargeastheotheris (a)-2/3
(b)1/3
(c)-1/3
(a)Geometric progression
(b)Harmonic progression
(c)Arithmetic - geometric progression
(d)Arithmetic progression
Answer: B
Solution:
Is/are true?
1 1 1 (𝑛+1)
(a) + +….,+ =
√𝑎1+√𝑎2 √𝑎2+√𝑎3 √𝑎𝑛−1+√𝑎𝑛 √𝑎1+√𝑎𝑛
1 1 1 (𝑛−1)
(b) + +….,+ =
√𝑎1+√𝑎2 √𝑎2+√𝑎3 √𝑎𝑛−1+√𝑎𝑛 √𝑎1+√𝑎𝑛
Q501. [MSQ]
Which of the following statement is/are true?
𝑦
(a) If𝑥√𝑎= 𝑏=√𝑧 𝑐andifa,b,careinG.P,thenx,y,zareinA.P.
√
(b) IfqthandrthtermsofanA.Pandbea,bandcrespectivelythen
𝑎 𝑏−𝑐 .𝑏 𝑐−𝑎 .𝑐 𝑎−𝑏=1.
(c) If𝑎 𝑥=𝑏 𝑦=𝑐 𝑧 andx,y,zareinG.P.,thenlog 𝑏𝑎=log𝑏.
𝑐
Directionsfornextfivequestions:Followingbarchartrepresentsthenumberofpeoplein6
differentvillages(A,B,C,D,EandF)andthetabularcolumndepictstheratioofliterateto
illiteratepeopleandpercentageofmalelivinginthosevillages.
Q504. What is the percentage of literate people in all the six villages together?
(a)55% (b)53% (c)51% (d)None of
theseAnswer:B
Solution:
Q505. What is the ratio between numbers of illiterate people from villages B, C & D to
number of female from villages A, E & F?
(a) 320: 527 (b) 527 : 330 (c) 330:527 (d) 527 :320
Answer: C
Solution:
Q507. The number of female from villages A & C is how much percentage more or less
than number of female from villages D & F?
(a)24.76% (b)24.72% (c) 25.76% (d)25.72%
Answer: A
Solution:
Q509. The total number of students clearing phase I from commerce and other department
is what percentage of the number of students clearing phase I from physics
department?
(a) 340 %
Q510. WhathastheratiobetweennumbersofstudentsappearedfromMathsandphysics
departments to the number of students appeared from chemistry and commerce
departments?
(a) 16 :15
(b) 15 :17
(c) 16 :17
(d) 15 : 16
Answer: C
Solution:
Q511. From which department is the difference between the number of students cleared
and the number of students appeared is the second minimum?
(a) Chemistry
(b) Physics
(c) Commerce
(d) Maths
Answer: A
Solution:
Directionsfornextfivequestions:Thefollowingpiechartshowsthedistancetravelledby
boatondifferentdays.Thespeedofthestreamissameinbothupstreamanddownstream
direction. The table shows the speed of stream at different days. Study the data given
carefullyandanswerthequestionsbasedonit.
Q513. On Wednesday the difference between speeds of the boat going downstream and
upstreamis7km/hrandalsotheratioofthespeedofboatgoingdownstreamand
upstream11:4.Ifthetotaltimetakenbyboattocoverdownstreamandupstream
distancesonWednesdayis26hours36minutes.Whatisthespeedofstreamonthat day?
(a)1kmph (b)1.5kmph (c)2kmph (d) 2.5kmph
Answer: A
Solution:
let speed of stream = v
Q514. IfthetotalspeedofaboatinstillwateronSundayandTuesdayis12.5km/hrand
alsotimetakentocoverthedownstreamdistanceonSundayisequaltothetime
takentocovertheupstreamdistanceonTuesday.Findtheratioofspeedinstill water on
Sunday andTuesday.
(a) 9: 16 (b) 16: 17 (c) 16:9 (d) 19 : 16
Answer: C
Solution: Let speed of boat on Sunday =u
Speed of boat on Tuesday = v
(u + v)=12.5km/hr
According to question,
(v+2)/(u-2.5)=(15%of360)/(18%of300)
(Time same so ratio of speed = ratio of distance)
v + 2 = u - 2.5
u – v = 4.5
u = 8, v = 4.5
Therefore u : v = 16 : 9
Option C is correct.
Q515. On Thursday the boat takes total 30 hours to cover upstream and downstream
distances.Iftheratioofthespeedoftheboatinstillwatergoingdownstreamand
upstreamis5:3.Findthespeedoftheboatinstillwaterwhilegoinginanupstream
direction.
(a) 8km/hr
(b) 12km/hr
(c) 6km/hr
(d) 19km/hr
Answer: C
Solution:
(25%of360)/(5x+2)+(30%of300)/(3x-2)=30
Q516. TimetakentocovertheupstreamdistanceonMondayis8hoursmorethanthetime
takentocoverthedownstreamdistanceonThursdayandtotalspeedinstillwater on
Monday and Thursday is 23 km/hr. What is the ratio of speed in still water on
Monday andThursday?
(a) 5 :18
(b) 7 :16
(c) 10 : 13
(d) 11 : 12
Answer:B
Solution: {20% of 300/ (u1-2) – 20% of 360/ (u2+2)} =8
u1+u2 =23
On solving u1= 7km/hr, u2 =
16km/hrSo option B is correct.
Q517. OnSundaytheratioofthespeedoftheboatinstillwatergoingupstreamand downstream
is 5:7. The difference between the time to cover upstream and
downstreamdistanceis6hours.Findthetotaltimetocovertheupstreamand
downstreamdistance.
(a) 6hours
(b) 12hours
(c) 18hours
(d) 15hours
Answer: C
Solution:
{12% of 300/(5x-2) – 15%of 360/(7x+2) }= 6
On solving x=1
Total time= 12+6= 18 hours
So option C is correct.
Q522. The percentage increase in UP from Wednesday to Thursday is what percent of the
percentage increase in Delhi from Tuesday to Wednesday?
(a)50% (b)25% (c) 45% (d)30%
Answer: D
Solution:
From common explanation, we have
44−90
Percentage increase in UP from Wednesday to Thursday = × 100 = 60%
90
Percentage increase in Delhi from Tuesday to Wednesday
108−36
= × 100 = 200%
36
Required % = 60 /200× 100 = 30%
From common
explanation, we have
Requireddifference=3600–2750=850
Hence, option D iscorrect.
Q527. The number of people who recovered in India are what percent of the people who
recovered in Spain?
(a) 5.83%
(b)7.14%
(c) 8.25%
(d) 6.67%
Answer: D
Solution:
SPATIAL APTITUDE
SERIES
Q1. Select a figure from amongst the Answer Figures which will continue the same series
as established by the five Problem Figures.
Q2. Select a figure from amongst the Answer Figures which will continue the same series
as established by the five Problem Figures.
(a) 1
(b) 2
(c) 3
(d) 4
Q3. Select a figure from amongst the Answer Figures which will continue the same series
as established by the five Problem Figures.
Q4. Select a figure from amongst the Answer Figures which will continue the same series
as established by the five Problem Figures.
Q5. Select a figure from amongst the Answer Figures which will continue the same series
as established by the five Problem Figures
Q6. Select a figure from amongst the Answer Figures which will continue the same series
as established by the five Problem Figures.
Q7. Select a figure from amongst the Answer Figures which will continue the same series
as established by the five Problem Figures.
Q8. Select a figure from amongst the Answer Figures which will continue the same series
as established by the five Problem Figures.
Q9. Select a figure from amongst the Answer Figures which will continue the same series
as established by the five Problem Figures.
(a) 20
(b)18
(c)16
(d)12
Q19. Count the number of triangles and squares in the given figure.
Q22. Find out from amongst the four alternatives as to how the pattern would appear
when the transparent sheet is folded at the dotted line.
Q24. Find out from amongst the four alternatives as to how the pattern would appear
when the transparent sheet is folded at the dotted line.
Q26. Find out from amongst the four alternatives as to how the pattern would appear
when the transparent sheet is folded at the dotted line.
Q28. Find out from amongst the four alternatives as to how the pattern would appear
when the transparent sheet is folded at the dotted line.
(a) 1
(b) 2
(c) 3
(d) 4
Q33. Find out from amongst the four alternatives as to how the pattern would appear
when the transparent sheet is folded at the dotted line.
(a) 1
(b) 2
(c) 3
(d) 4
(a) 1
(b) 2
(c) 3
(d) 4
Q35. Find out from amongst the four alternatives as to how the pattern would appear
when the transparent sheet is folded at the dotted line.
(a) 1
(b) 2
(c) 3
(d) 4
Q41. Select a suitable figure from the Answer Figures that would replace the question
mark (?).
Problem Figures:
(a) 1
(b) 2
(c) 3
(d) 4
Q43. Select a suitable figure from the Answer Figures that would replace the question
mark (?).
(a) 1
(b) 2
(c) 3
(d) 4
Q52. Choose the correct mirror image of the given figure (X) from amongst the four
alternatives.
Q53. Choose the correct mirror image of the given figure (X) from amongst the four
alternatives.
(a) 1
(b) 2
(c) 3
(d) 4
Q67. Choose a figure which would most closely resemble the unfolded form of Figure (Z).
(a) 1
(b) 2
(c) 3
(d) 4
(a) 1
(b) 2
(c) 3
(d) 4
Q69. Choose a figure which would most closely resemble the unfolded form of Figure (Z).
(a) 1
(b) 2
(c) 3
(d) 4
Q70. Choose a figure which would most closely resemble the unfolded form of Figure (Z).
(a) 1
(b) 2
(c) 3
(d) 4
Q81. Choose the box that is similar to the box formed from the given sheet of paper (X).
Q84. Choose the box that is similar to the box formed from the given sheet of paper
(X).Choosetheboxthatissimilartotheboxformedfromthegivensheetofpaper(X).
(a) 1only
(b) 2 and 3only
(c) 1 and 3 only
(d)1, 2 and 4only
Q88. When the following figure is folded to form a cube, how many dots lie opposite the
face bearing five dots?
(a) 1
(b) 2
(c) 5
(d) 6
Q90. What will be the number at the bottom, if 5 is at the top; the two positions of the dice
being as given below
Q97. Choose the alternative which is closely resembles the water-image of the given
combination.
Ineachofthefollowingquestions,choosethewaterimageoftheFig.(X)fromamongstthe
fouralternatives(1),(2),(3)and(4)givenalongwithit
Q99. Choose the correct water image of the given figure (X) from amongst the four
alternatives.
Q101. Choose the correct water image of the given figure (X) from amongst the four
alternatives.
(a) 1
(b) 2
(c) 3
(d) 4
(a) 1
(b) 2
(c) 3
(d) 4
(a) 1
(b) 2
(c) 3
(d) 4
(a) 1
(b) 2
(c) 3
(d) 4
Q2. Some nouns are always used in a plural form and always take a plural verb. Trousers,
scissors,spectacles,stockings,shorts,measles,goods,alms,premises,thanks,tidings,
annals, chattels,etc
a) Where ismytrousers? (Incorrect)
b) Where aremytrousers? (Correct)
a) Spectacles is now acostlyitem. (Incorrect)
b) Spectacles are now acostlyitem. (Correct)
Q3. Collective nouns such as jury, public, team, committee, government, audience,
orchestra, company, etc. are used both as singular and plural depending on the
Meaning. When these words indicate a unit, the verb is singular, otherwise the verb
will be plural.
a) The jury was divided inthiscase. (Incorrect)
b) The jury were divided inthiscase. (Correct)
a) The team have not comeasyet. (Incorrect)
b) The team has not comeasyet. (Correct)
Q5. A pronoun must agree with its antecedent in person, number and gender.
For example:
Every man must bring his luggage.
All students must do their home work.
Each of the girls must carry her own bag.
Each student must bringtheirbooks. (Incorrect)
Each student must bring hisbooks. (Correct)
Q8. „Who‟ denotes the subject and „whom‟ is used for the object?
Whom do you think wontheaward? (Incorrect)
Who do you think wontheaward? (Correct)
Who are youtalkingto? (Incorrect)
Whom are youtalkingto? (Correct)
Q9. When two or more singular nouns are joined together by „either or‟; „neither nor‟;and
„or‟, thepronounis singular.
Either Ram or Shyam will givetheirbook. (Incorrect)
EitherRamorShyamwillgivehisbook. (Correct)
Q10. „Whose‟isusedforlivingpersonsand„which‟forlifelessobjects?
Which book did youselect?
Whose photograph is lying there?
What book doyou read? (Incorrect)
Which book doyouread? (Correct)
Q12. Whenapronounstandsforacollectivenoun,itmustbeinthesingularnumberand
intheneutergenderifthecollectivenounisviewedasawhole.
The jury gave „its‟ verdict.
Herethe„jury‟givestheideaofonewhole.
If the collective noun conveys the idea of separate individuals comprising the whole,
the pronoun standing for it must beplural.
The jury were divided in their opinions.
Here the „jury‟ gives the idea of several individuals.
The team are divided in this opinion about playingonSunday. (Incorrect)
The team are divided in their opinion about playingonSunday. (Correct)
Q13. If pronoun of different persons are to be used together in a sentence, the serial order
of persons should be as follows: second person + third person + first person in a good
normal sentence. But if a fault is to be confessed, the order will be:
first person + second
person + third person.
You, he and I have finishedthework. (Normalsentence)
I, you and he aretoblame. (Confession)
Ram, I and you have finishedour studies. (Incorrect)
You, Ram and I have finishedourstudies. (Correct)
Q19. Normally „than‟ is used in the comparative degree, but with words like superior,
Inferior,senior,junior,prior,anterior,posteriorandprefer„to‟isused.Shelleyisjunior
thanWordsworth. (Incorrect)
Shelley is juniortoWordsworth. (Correct)
I prefer readingthansleeping. (Incorrect)
I prefer readingto sleeping. (Correct)
Q22. If the subject is „the number of‟ the singular verb is used.
The number of students arevery small. (Incorrect)
The number of students isverysmall. (Correct)
Ifthesubjectis„anumberof‟,thepluralverbisused.
A number of booksismissing. (Incorrect)
A number of booksaremissing. (Correct)
Q24. „A great many‟ is always followed by a plural noun and a plural verb.
A great many student has beendeclaredsuccessful. (Incorrect)
A great many students have beendeclaredsuccessful. (Correct)
Q26. When two singular nouns joined by „and‟ point out the same thing or person, the verb
will be singular.
Bread and butter make agoodbreakfast. (Incorrect)
Bread and butter makes agoodbreakfast. (Correct)
The Collector and the District Magistrate areonleave. (Incorrect)
TheCollectorandDistrictMagistrateisonleave. (Correct)
Q28. Use of „not only‟ and „but also‟: Examine the sentences given below.
He not only comes for swimming but also for coachingthelearners. (Incorrect)
He comes not only for swimming but also for coachingthelearners. (Correct)
Here the error is due tomisplacement.
Theexpression„notonly‟mustbeplacedbeforetherightword,otherwisethemeaning
changes. Here this expression is meant for swimming and not forcoming.
Q29. „Scarcely‟ and „hardly‟ are followed by „when‟ and not by „than‟.
I had scarcely entered the room than thephonerang. (Incorrect)
I had scarcely entered the room when thephonerang. (Correct)
Q34. „Unless‟ expresses a condition. It is always used in the negative sense. Thus „not‟ is
never used with „unless‟.
Unless you do not labour hard, you willnotpass. (Incorrect)
Unless you labour hard, you willnotpass. (Correct)
Q40. Two actions in the past, one depending on the other, should have the sequence as
follows:
Past perfect + Future perfect
Had + past participle + would + have + past participle
If you had worked hard, you would have succeeded in the examination.
Or
Had you worked hard, you would have succeeded in the examination.
If you would have practised regularly, you would winthematch. (Incorrect)
If you had practised regularly, you would have wonthematch. (Correct)
Q41. If, in a sentence, two actions are indicated and both are to take place in future, the
sequence of tenses will be as follows:
The principal clause in present indefinite; and the subordinate clause in future
indefinite.
If I go to Delhi, I shall attend the seminar.
„If I go to Delhi‟ is the principal clause and „I shall attend the seminar‟ is the
subordinate clause.
If it will rain, I shall not attendthemeeting. (Incorrect)
If it rains, I shall not attendthemeeting. (Correct)
Q42. When an action has taken place in two clauses of a sentence, it is used in both the
clauses according to the requirement.
My brother has and is still doing excellent work forhisorganisation. (Incorrect)
My brother has done and is still doing excellent work for his organisation. (Correct)
Q45. Similarly, words like „hour‟, „honest‟, „heir‟, etc. take „an‟ before them as they begin
with a vowel sound.
I have been waiting for him forahour. (Incorrect)
I have been waiting for him foranhour. (Correct)
Q48. „What to speak of‟ is incorrect; the correct expression is „not to speak of‟.
What to speak of running, he cannotevenwalk. (Incorrect)
Not to speak of running, he cannotevenwalk. (Correct)
Q50. „Cent per cent‟ and „word by word‟ are wrong. „Hundred per cent‟ and „word for
word‟ are correct expressions.
You are never cent per cent sure of your success in a competitive examination.
(Incorrect)
You are never hundred per cent sure of your success in a competitive examination.
(Correct)
I can reproduce this lesson word by word. (Incorrect)
I can reproduce this lesson word for word.(Correct)
Q51. „Since‟,„because‟,„as‟and„for‟–allmean„because‟,butthereisadifferenceintheir
degree.„Since‟and„because‟areusedforstrongercasesand„as‟and„for‟forweak cases.
I respect him as he ismyteacher. (Incorrect)
I respect him because he ismyteacher. (Correct)
I could not attend the meeting because it was veryhottoday. (Incorrect)
I could not attend the meeting as it was veryhottoday. (Correct)
Q52. Use of „when‟ and „while‟: Proper attention must be paid to these words. „When‟
indicatesageneralsenseand„while‟impliesatimeduringtheprocessofdoingawork. When
learning to swim, one of the most important things istorelax. (Incorrect)
While learning to swim, one of the most important things istorelax. (Correct)
Q54. Proper, abstract and material nouns have no plural except when they are used as
common nouns.
The house is builtofbricks. (Incorrect)
The house is builtofbrick. (Correct)
When such nouns are used in the plural, they become common nouns with changed
meanings; as
Coppers =Coppercoins; Irons =fetters.
Tins = cans madeoftin; Woods =forest.
Helivesinthewoods.Itmeanshelivesinaforest.
Q55. The relative pronoun should be placed as near as possible to its antecedent so that no
ambiguity arises.
The boy is my cousin who stood first in themilerace. (Incorrect)
Theboy,whostoodfirstinthemilerace,ismycousin. (Correct)
Q56. When two singular nouns joined by „and‟ are preceded by „each‟ or „every‟ the
pronoun used for them is singular.
Each man and each boy is responsible fortheiraction. (Incorrect)
Each man and each boy is responsible forhisaction. (Correct)
Q59. When a pronoun comes after „like‟ and „unlike‟ it takes an objective case.
A man like I will notdoit. (Incorrect)
A man like me will notdoit. (Correct)
Q60. A pronoun takes an objective case after „let‟.
Let Idoit. (Incorrect)
Let medoit. (Correct)
Q62. When pronouns of second person and third person are used as subjects, the pronoun
Following them will be according to the second person pronoun.
You and he must bringhisbooks. (Incorrect)
You and he must bringyourbooks. (Correct)
Q63. When pronouns of second person and first person are used as subjects, the pronoun
following them will be first person plural form.
You and I must finish your workintime. (Incorrect)
You and I must finish our workintime. (Correct)
Q64. „But‟ is also used as a relative pronoun. When a sentence has a negative noun or
pronoun, „but‟ can be used with it. In this case „but‟ means: who not; that not.
Here was none but laughed to see the joker.
Here the meaning of „but laughed‟ is „who did not laugh.‟
There is no bird but flies.
„But flies‟ means „that does not fly‟.
Q68. Whentherearetwoantecedents,amanandananimalortwothingsbeforetherelative
pronoun, we should use„that‟.
The man and his dog which passed through this roadwerekilled. (Incorrect)
The man and his dog that passed through this roadwerekilled. (Correct)
Q69. The case of the noun or pronoun preceding or succeeding the verb „to be‟ should be
the same.
It is him who came toseeus. (Incorrect)
It is he who came toseeus. (Correct)
It is me who caughtthethief. (Incorrect)
It is I who caughtthethief. (Correct)
Q71. When two or more adjectives are used to show the qualities of the same man or thing,
all the adjectives must be in the same degree.
Sita is more intelligent and wisethanRita. (Incorrect)
Sita is more intelligent and wiserthanRita. (Correct)
Ramesh is the wisest and strong boy oftheclass. (Incorrect)
Ramesh is the wisest and the strongest boy oftheclass. (Correct)
Q72. „Very‟ is used with adjectives in the positive degree and with present participles.
He is a muchstrong man. (Incorrect)
He is a verystrongman. (Correct)
It is a muchinterestingbook. (Incorrect)
It is a veryinterestingbook. (Correct)
„Much‟ is used with adjective in the comparative degree and with past participles.
He is very stronger thanIam. (Incorrect)
He is much stronger thanI am. (Correct)
I am very obliged tomyfriend. (Incorrect)
I am much obliged tomyfriend. (Correct)
Q73. To show equality „as‟ is used before and after the adjective.
I can run as fast, if not faster,thanyou. (Incorrect)
I can run as fast as, if not faster,thanyou. (Correct)
Q77. If the plural subject denotes a definite amount or quantity taken as a whole, the verb
is singular.
Forty miles are agooddistance. (Incorrect)
Forty miles is agooddistance. (Correct)
Two-thirds of the bookwererubbish. (Incorrect)
Two-thirds of the bookwasrubbish. (Correct)
Q80. The following verbs and phrases should be followed by a gerund which is a verbal
noun:„enjoy‟,„admit‟,„deny‟,„appreciate‟,„accustomedto‟,„isusedto‟,„donotmind‟,
„object to‟,etc.
I am looking forward to receiveyourreply. (Incorrect)
I am looking forward to receivingyour reply. (Correct)
He is used toworkhard. (Incorrect)
He is used toworkinghard. (Correct)
Q81. The word „to‟ is frequently used with the infinitive, but it is not an essential part ofit.
Forexample,aftercertainverbs(„bid‟,„let‟,„make‟,„need‟,„dare‟,„see‟,„hear‟)weuse the
infinitive without„to‟.
Bid him togothere. (Incorrect)
Bid himgothere. (Correct)
Make himtostand. (Incorrect)
Makehimstand. (Correct)
Q82. The infinitive is used without „to‟ after „had better‟, „had rather‟, „would rather‟,
„sooner than‟ and „rather than‟.
You had better to ask permissionfromhim. (Incorrect)
You had better ask permissionfromhim. (Correct)
Q2. (A)Theprettywoman/(B)sittinginthecaris/(C)thedaughterof/(D)theoneof
Q3. (A)Youareamongthose/(B)luckiestmanwho/(C)attainednameandfamenotby
Q4. (A)TheBoardofDirectors/(B)wantallpossiblefacilities/(C)andallowancesfor/
Q5. (A)YouandI/(B)havedone/(C)mybestin/(D)theexamination./(E)Noerror.
Q6. (A) His efforts /(B) will bring him /(C) more success /(D) than your. /(E) No error.
Q7. (A)Everyteacherand/(B)everystudentofthiscollege/(C)isdeterminedtodotheir
best for /(D) the glorious prospects of the college. /(E) No error.
Q8. (A)TheVice-PresidentofIndiaand/(B)theVice-Chancellorofthisuniversity/(C)
have given /(D) his consent to join the meeting. /(E) No error.
Q9. (A) She hates everybody /(B) and everything who /(C) reminds her /(D) of her
mistakes. /(E) No
Q10. (A)None/(B)ofthesetwoPrincipals/(C)hasbeenlooking/(D)afterhiscollegewell.
/(E) No error.
Q11. (A)Oneshould/(B)betruetohis/(C)wordinall/(D)circumstances./(E)Noerror.
Q14. (A)Asastudent/(B)ofscience/(C)youarefarbetter/(D)thanhim./(E)Noerror.
Q15. (A)Thisisthesame/(B)dogwhichbarked/(C)athimbutfortunatelydidnot/(D)
Q16. Whomever /(B) comes late will not be allowed /(C) to make his presence /(D) in the
Q17. Youandmyself/(B)willenjoytheparty/(C)tobearrangedinhonourof/(D)thenew
Q18. (A) You say /(B) it‟s your problems /(C) but I say it‟s /(D) my also. /(E) No error.
Q19. (A) The hotels /(B) of Patna are more /(C) costly than /(D) Muzaffarpur. /(E) No
error.
Q20. (A)Shewasmore/(B)garrulousthan/(C)eitherofher/(D)threesisters./(E)No
error.
Q21. (A) When you will find out /(B) any solution to this problem, /(C) you will become
Q22. (A)Whenever/(B)youarecominghere,/(C)youbringalotof/(D)sweetsforme.
/(E) No error.
Q23. (A)Itisappearingtome/(B)thatyouareplotting/(C)againstyour/(D)friendsand
Q2. (A) The secret of his good /(B) health lies in the fact that /(C) he is getting up before
Q3. (A) She says /(B) that she will take /(C) her umbrella /(D) in case it will rain. /(E)
No error.
Q4. (A) Before the alarm /(B) had stopped ringing /(C) Veera had pulled up /(D) the
Q5. (A) I have been knowing /(B) him for ten years /(C) but I don‟t know /(D) wherehe
Q6. (A) All the flowers /(B) in his garden will surely die /(C) before the rains /(D) will
Q7. (A) If I would have done this, /(B) I would have done wrong /(C) and would have
Q8. (A) I wish /(B) he saw you /(C) when you were /(D) living in England. /(E) No
error.
Q9. (A)Thejudgehadnotgiven/(B)anydicision/(C)untilhehad/(D)studiedthe
Q10. (A) Radha came /(B) to the meeting /(C) much later /(D) than I expect. /(E) No
error
Q11. (A) If I was you /(B) I would have told /(C) the Chairman to keep /(D) his mouth
shut. /(E) No error.
Q13. (A) The problems that our /(B) country has and is bound to face /(C) are the
Q14. (A) Never before /(B) the students of India have so much been frustrated /(C) as
today when the problems /(D) of unemployment are dominant. /(E) No error.
Q15. (A)Beingarainyday/(B)Vijaydecidedtostay/(C)athomeandwork/(D)further
Q16. (A)Schooloffersmanyopportunitiesofmeeting/(B)helpfulpeople,readinguseful
books/(C)andobtaininformationabout/(D)avarietyofpubliccareers./(E)No error.
Q17. (A) Taking tea, /(B) he went to the office /(C) and disposed /(D) of a lot of work.
/(E) No error.
Q18. (A) Only one dispute /(B) has been settled /(C) and others left to both the parties
Q19. (A) Without taking proper care, /(B) the doctors could not have been saved /(C)this
patient who received /(D) a bullet injury in the chest. /(E) No error.
Q20. (A) Going towards them /(B) with some cups of /(C) tea in the tray, somebody
Q21. (A) During the course of investigation /(B) the police learnt that not only /(C) the
house was looted but /(D) also the things taken away. /(E) No error.
/(E) No error.
Q23. (A)Weadvisedhim/(B)tomarryhisdaughter/(C)becauseshehad/(D)cometo
Q24. (A) The criminal /(B) will certainly be hung /(C) because the charges brought upon
Q25. (A) Owing to his /(B) ill health, he will /(C) not be able to give this examination,
which means /(D) he will have to lose one year. /(E) No error.
Q2. (A) When she entered /(B) the room, she found the /(C) child sleeping peacefully
Q3. (A) Two miles beyond /(B) that pasture was hundreds /(C) of cattle including /(D)
Q4. (A)Itisnoticedthat/(B)theeliteclasshavenosoft/(C)cornerforthedowntrodden
who are the /(D) real victims of the present social set-up. /(E) No error.
Q5. (A) Ritu as well as /(B) some of her friends /(C) have fallen in love with Sonu who
Q6. (A) More than one successful candidate /(B) have given interviews for one /(C) of
the popular magazines /(D) being published from Delhi. /(E) No error.
Q7. (A) The leader as well as /(B) his followers are fatally injured in /(C) the train
accident which occurred last night /(D) near this railway crossing. /(E) No error.
Q8. (A) Just outside /(B) the hotel is /(C) two bars, extremely beautiful /(D) as well as
crowdy. /(E) No .
Q9. (A) We should not forget that /(B) we have a right to criticize but /(C) at the same
time each of us /(D) have to remember the duty also. /(E) No error.
Q10. (A)Oneof/(B)thebiggestindustrialhouses/(C)inBombayisonthevergeof/(D)
declaringalockout./(E)Noerror.
programmestobeperformed/(D)intheCityHall,areuptothemark./(E)Noerror.
Q12. (A) Everybody among the businessmen /(B) were injoying drinking /(C) when the
Managerofthehotel/(D)wasshotdead./(E)Noerror.
Q13. (A) Although these buildings are /(B) in need of repair, /(C) there have been much
Q14. (A)Notonlythedoctor/(B)butalsothenursesofthisnursing/(C)homeisverykind
Q15. (A)Eachfacultymemberas/(B)wellasmostofthestudents/(C)wereoftheview
that there should /(D) be many more new courses. /(E) No error.
Q16. (A) Either the manager /(B) or his assistants always try to misguide the public /(C)
Q17. (A)Everymanandwoman/(B)ofthevillagehavecomeout/(C)toseethisstrange
child/(D)whoclaimstoknoweverythingabouthispre-natalexistence./(E)Noerror.
Q18. In our college, it /(B) was obligatory for each of /(C) the students to buy /(D) his own
Q19. (A) One of the developing or underdeveloped /(B) countries are in favour of
launching/(C)chemicalwarfarebecauseittrembles/(D)toimagineitsrepercussions.
/(E) No error.
Q20. (A)NeitherRajni/(B)norRaginiweretotaketoherheels/(C)whentheysawacobra
Q22. (A)Wassheabird/(B)shewoulddefinitelyflytoyou/(C)andsaythatshecouldnot
Q23. (A) He was not /(B) so well versed in /(C) English that we /(D) had expected. /(E)
No error.
Q24. (A) Mrs. Varun /(B) not only stopped coming /(C) here but also going to any place
Q25. (A) Neither the doctor /(B) nor his assistants /(C) were asleep when /(D) the phone
EXERCISE-4
Q1. (A)Youmusteither/(B)informthepolice/(C)elsebeprepared/(D)tosufferanyloss.
/(E) No error.
Q2. (A) As soon as the peon /(B) rings the first bell, then all /(C) the students assemble
Q3. (A) He not only comes /(B) here for shopping but also for /(C) having a glimpse of
Q4. (A) Hardly had he /(B) come out of the bus /(C) then the bomb exploded /(D) and
Q5. (A) Scarcely had he bought /(B) the ticket when the guard /(C) showed the flag and
Q7. (A)Thisisperhaps/(B)thesamedogwhich/(C)bitherwhileshewasreturning/(D)
Q8. (A) Both Hari /(B) as well as his wife /(C) are determined to bring /(D) about some
changesintheplan./(E)Noerror.
Q9. (A)Asheis/(B)aperfectionist,/(C)sohealwaysinsists/(D)onregularpractice./
(E) No error.
Q10. (A)Althoughtheserooms/(B)areinneedofrepair,/(C)buttheownerdoesnottake
Q11. (A)Hesuggestedme/(B)thatIshould/(C)doalltheworkas/(D)quicklylikehim.
/(E) No error.
Q12. (A)Nosoonerdid/(B)hefindoutanysolution/(C)totheproblemwhen/(D)another
Q13. (A) The robbers had /(B) hardly put the ornaments /(C) in his bag than /(D) the
housewifewokeup./(E)Noerror.
Q14. (A)Scarcelyhadhe/(B)goneafewsteps/(C)thathewastoldbysomeone/(D)that
Q15. (A) Mrs. Sen told me that though /(B) her son had worked hard but /(C) he failed to
Q16. (A) Dr. Sinha was not only /(B) sympathetic to the rich /(C) patients but also /(D) to
Q18. (A) Both the rich along with /(B) the poor are responsible for a /(C) great many vices
with which our society /(D) as well as country is inflicted. /(E) No error.
Q19. (A) He exclaimed with sorrow /(B) that his brother died /(C) just two /(D) months
before./(E)NoError.
Q20. (A)InsteadofgoingtoLondon/(B)hewenttoAmerica/(C)andstays/(D)therefor
Q21. (A)Theprisonerswalkedslowly/(B)fortheyknewthatas/(C)soonastheycrossthe
gate, /(D) the jailor would ask them to jog. /(E) No error.
Q22. (A)Theprisonerswalkedslowly/(B)fortheyknewthatas/(C)soonastheycrossthe
gate, /(D) the jailor would ask them to jog. /(E) No error.
Q23. (A)WhenMeenasaid/(B)thatshewascomingtosee/(C)methenextday,Iwondered
Q24. (A) I suggested that Ragini should /(B) stay here at night if she /(C) got late but she
Q25. (A)Heaskedme/(B)whyhadIgonetotheCinema/(C)lateatnightinspiteof/(D)
drawing room.
Q2. (A)Herequestedtheteacher/(B)toallowhimtogohome/(C)ashegot/(D)asevere
Q3. (A) Jyoti asked me /(B) what could she do /(C) for me in that /(D) critical situation.
/(E) No error.
Q4. (A) The Director knowing of my /(B) interest in Environmental Science /(C) asked me
that I would /(D) like to attend the National Seminar. /(E) No error.
Q5. (A)TheGovernmentwarnedtheshopkeepersthatif/(B)theypersistin/(C)charging
Q6. (A)Iwassurprised/(B)toknowwhyhadheturned/(C)downsuchagoodoffer/(D)
of marriage.
Q7. (A) My father wanted to know /(B) that I had done /(C) all the work /(D) assigned to
me./(E)NoError.
Q8. (A) In his famous sonnet, /(B)Milton says that /(C) “They also serve /(D) who only
Q9. (A) Any step that the centre /(B) takes to establish /(C) peace and harmony in the
Q10. (A)Thepolicemen/(B)whowereondutyinthisarea/(C)werediscovered/(D)two
Q12. (A) The teacher asked /(B) the students whether they could /(C) tell the name of the
Q13. (A) She held something /(B) at her side which /(C) was totally hiding /(D) by the folds
Q14. (A) I was surprising /(B) at not having seen /(C) her even though she was standing
Q15. (A)Incaseyouapologise/(B)forhavingbrokenyour/(C)promisesyouwill/(D)
Q16. (A)Hadthepolicenot/(B)reachedhere/(C)ontime,Iwould/(D)havekilledbythe
robbers/(E)Noerror.
Q17. (A)Bywhom/(B)washehelped/(C)whenheseriouslywounded/(D)byaterrorist?
/(E)Noerror.
Q18. (A)APoliceInspectorsent/(B)tomyhouse/(C)andalltherooms/(D)werechecked.
/(E)Noerror.
Q19. (A)Theneedyman/(B)wasapproachedtothemoneylender/(C)andrequestedhim
to/(D)lendhimsomemoney./(E)NoError.
Q20. (A)Itwassuggested/(B)bythedoctor/(C)thatthepatient/(D)shouldbetakencare.
/(E)NoError.
Q21. (A) She was extremely /(B) annoyed by /(C) her lover‟s unreasonable /
(D)behavior./(E)NoError.
whichtookplacelastnight./(E)NoError.
Q23. (A)Whenhelentme/(B)somebooks,heasked/(C)metoreturnthemback/(D)by
theendofthemonth./(E)NoError.
EXERCISE-6
Q1. (A)Hedenied/(B)thathewasnot/(C)presentatthespotof/(D)occurrence./
(E) No Error.
Q2. (A)Myfatheraskedme/(B)ifIhad/(C)sufficientenoughmoney/(D)topaythefees.
/(E)NoError.
Q3. (A)Thecandidate/(B)requestedthechairman/(C)torepeatthe/(D)questionagain.
/(E)NoError.
Q4. (A)Theprincipalforbade/(B)thestudentsnotto/(C)stagea/(D)walkout./€No
Error.
Q5. (A) “The patient is comparatively better /(B) today and I hope /(C) that he will
recoversoon”,/(D)saidthedoctor./(E)NoError.
Q6. (A)Sheaskedmepolitely/(B)whereIwasgoingto/(C)andwhatIwould/(D)bring
forher./(E)NoError.
Q7. (A)Probablybytheend/(B)oftheyearhewill/(C)leavefortheUSA/(D)withbag
andbaggage./(E)NoError.
Q8. (A) Throughout the whole year /(B) there was not /(C) a single day /(D) without
someviolence./(E)NoError.
Q9. (A)Shereimbursedback/(B)themoneywhichIhad/(C)spentduringour/(D)
journeyto Kathmandu./(E)NoError.
Q2. COMMISSIONED
(a)Started (b)Closed (c)Finished (d)Terminated
Q3. ARTIFICIAL
(a)Red (b)Natural (c)Truthful (d)Solid
Q4. EXODUS
(a)Influx (b)Home-coming (c)Return (d)Restoration
Q5. RELINQUISH
(a)Abdicate (b)Renounce (c)Possess (d)Deny
Q6. EXPAND
(a)Convert (b)Condense (c)Congest (d)Conclude
Q7. MORTAL
(a)Divine (b)Immortal (c)Spiritual (d)Eternal
Q8. QUIESCENT
(a) ACTIVE (b)Dormant (c)Weak (d)Unconcerned
Q9. OBEYING
(a) Ordering (b)Following (c) Refusing (d) Contradicting
Q10. FRAUDULENT
(a)Candid (b)Direct (c) Forthright (d)Genuine
Q2. BRIEF
(a)Limited (b)Small (c)Little (d)Short
Q3. EMBEZZLE
(a) Misappropriate (b)Balance (c)Remunerate (d)Clear
Q4. VENT
(a)Opening (b)Stodge (c)End (d)Past tense of go
Q5. AUGUST
(a) Common (b) Ridiculous (c) Dignified (d)Petty
Q6. CANNY
(a) Obstinate (b)Handsome (c)Clever (d)Stout
Q7. ALERT
(a) Energetic (b) Observant (c)Intelligent (d)Watchful
Q8. WARRIOR
(a) Soldier (b)Sailor (c)Pirate (d)Spy
Q9. DISTANT
(a)Far (b)Removed (c)Reserved (d)Separate
Q10. ADVERSITY
(a) Failure (b)Helplessness (c)Misfortune (d)Crisis
(b) To ruinoneself
(d)To overcomesomeone
Read the following information carefully and answer the question given below it :
Q27. A watch reads 4.30. If the minute hand points East, in what direction will the hour
hand point?
(a) South-East
(b) North-East
(c) North
(d)North-West
Answer:B
Q28. „A‟ walks 10 m, towards East and then 10 m to his right. Then every time turning to
his left, he walks 5, 15 and 15 m, respectively. How far is he now from his starting
point?
(a)5m (b)10m (c)15m (d) 20m
Answer:A
Q29. A postman was returning to the post office which was in front of him to the north.
When the post office, was 100 m away from him, he turned to the left and moved 50
mtodeliverthelastletterattheShantivilla.He,thenmovedinthesamedirectionfor 40 m,
turned to his right and moved 100 m. How many metres was he away from the
postoffice?
(a)0m (b)150m (c)90m (d) 100m
Answer:C
BLOOD RELATION
Q33. IfA+BmeansAisthemotherofB;A-BmeansAisthebrotherB;A%BmeansA
isthefatherofBandAxBmeansAisthesisterofB,whichofthefollowingshows that P is
the maternal uncle ofQ?
(a) Q - N + Mx P (b) P + S x N -Q
(c) P - M + NxQ (d) Q - S %P
Q34. Introducing a man, a woman said, 'He is the only son of my mother's mother." How
is the woman related to the man?
(a)Mother (b)Aunt (c)Sister (d)
NieceAnswer:D
Q41. Lookingataportraitofaman,Sanjaysaid,“Hismotheristhewifeofmyfather‟sson.
Brothers and sisters I have none.” At whose portrait was Sanjay looking?
Q42. Old man‟s son is my son‟s uncle, then what relation has the old man to me?
(a)Brother (b)Father
(c)Grandfather (d)Uncle
Q43. If(1)„S+K‟means„SisthesisterofK‟;(2)„S–K‟means„SisthefatherofK‟;(3)„S× K‟ means „S
is the brother of K‟; which of the following means „P is the aunt of D‟? (a)P + M–D
(b)P – M ×D
(c)P – M+D (d)P × M –D
Directions (Q.45 – 46): Read following statements carefully and answer questions given
below:
(A) P×QmeansPisthebrotherofQ
(B) P+QmeansPisthefatherofQ
(C) P÷QmeansPisthesisterofQ
Q44. Which of the following represents A is uncle of M?
(a) A + D÷M (b)A × D +M
(c) A + D×M (d)A ÷ D +M
Q45. Which of the statements is superfluous to answer the above question?
(a)Conly (b)B or Conly
(c)Aonly (d)None ofthese
Q46. P is brother of Q. R is sister of Q. How P is related to R?
(a)Uncle (b)Datainadequate
(c)Brother (d)None ofthese
Q47. If P $ Q means P is father of Q, P # Q means P is mother of Q, P * Q means P is sister
of Q, then how is Q related to N in N # L $ P *Q?
(a)Noneofthese (b)Granddaughter
(c)Nephew (d)Datainadequate
Q48. A$B means B is father of A, A#B means B is mother of A, A*B means B is sister of A,
A@B means B is husband of A, then which of the following indicates the relationship
„N‟ is grandmother of Q?
Directions (Q.51 – 52): Study the following information carefully and answer the given
questions based on it.
i. „P × Q‟ means „Q is motherofP‟ ii. „P + Q‟ means „P is father ofQ‟.
iii. „P – Q‟ means „P is brotherofQ. iv. „P † Q‟ means „Q is sister ofP‟.
Q51. Which of the following statements is superfluous to answer the above questions?
(a)i only
(b) iionly
Directions (Q.53–54): Study meaning of given symbols and answers questions based on it.
i. „P×Q‟means„QismotherofP.
ii. „P + Q‟ means „P is brother ofQ‟.
iii. „P – Q‟ means „P is sister ofQ‟.
iv. „P † Q‟ means „Q is father ofP‟.
Q52. Which of the following definitely means R is grandson of K?
(a)R × T ÷ K
(b)R + M × T ÷ K
Q53. Which of the following statements is superfluous to answer the above questions?
(a)ivonly
(b)i only
(c)ii only
(d)iiionly
Directions (Q.56 – 57): Study the following information carefully and answer the given
questions following it.
i. „P × Q‟ means „P is brother ofQ‟.
ii. „P + Q‟ means „Q is mother ofP‟.
iii. „P – Q‟ means „Q is sister ofP‟
iv. „P † Q‟ means „P is father ofQ‟.
Q55. Which of the following means T is nephew of R?
(a)R × J ÷ T
(b) R – M + T – J
(c)R × J ÷ T × K
(d) None of these
Q56. Which of the following means is superfluous to answer the above questions?
(a)ionly (b)iionly
ANALYTIC PUZZLES
Directions (Q.1 – 4): Read the following information and answer the questions givenbelow
it:SixstudentsA,B,C,D,EandFaresittinginthefield.AandBarefromNehruHousewhile
therestbelongtoGandhiHouse.DandFaretallwhiletheothersareshort.A,CandDare
wearing glasses while the others are not.
Q59. Which two students, who are not wearing glasses are short?
(a) AandF (b)CandE (c)BandE (d) E andF
Directions(Q.64–68):Readfollowinginformationcarefullyandanswerthequestionsgiven
belowit:TherearesixpersonsA,B,C,D,EandFinaschool.Eachoftheteachersteachestwo subjects,
one compulsory subject and the other optional subject. D‟s optional subject was
Historywhilethreeothershaveitascompulsorysubject.EandFhavePhysicsasoneoftheir
subjects.F‟scompulsorysubjectisMathematicswhichisanoptionalsubjectofbothCandE. History
and English are A‟s subjects but in terms of compulsory and optional subjects, they
arejustreverseofthoseofD‟s.Chemistryinanoptionalsubjectofonlyoneofthem.Theonly
female teacher in the school has English as her compulsory subject.
Q63. What is C‟s compulsory subject?
(a) History (b)Physics
(c)Chemistry (d) English
Q65. Which of the following has same compulsory and optional subjects as those of F‟s?
(a)D (b)B (c)A (d)C
Q66. Disregarding which is the compulsory and which is the optional subject, who has the
Same two subject combination as F?
(a)A (b)B (c)E (d)D
Q67. Which of the following groups has History as the compulsory subject?
(a) A,C,D (b) B,C,D (c)C,D (d)A, B,C
Q69. Five children were administered psychological tests to know their intellectual levels.
In the report, psychologists pointed out that the child A is less intelligent than the
child B. The child C is less intelligent than the child D. The child B is less intelligent
than the child C and child A is more intelligent than the child E. Which child is the
most intelligent?
(a)A (b)B (c)D (d)E
Q71. Five boys participated in a competition. Rohit was ranked lower than Sanjay. Vikas
wasrankedhigherthanDinesh.Kamal‟srankwasbetweenRohitandVikas.Whowas
rankedhighest?
(a)Sanjay (b)Vikas (c)Dinesh (d)Kamal
Q72. Ashish is heavier than Govind. Mohit is lighter than Jack. Pawan is heavier than Jack
but lighter than Govind. Who among them is the heaviest?
(a)Govind (b)Jack (c)Pawan (d)Ashish
Q73. Rohan is taller than Anand but shorter than Seema. Krishna is taller than Pushpa but
shorter than Anand. Dhiraj is taller than Krishna but shorter than Seema. Who among
them is the tallest?
Q75. Which of the given information is not necessary to answer the above question?
(a)A (b)B (c)C (d)D
Q76. BistwiceasoldasAbuttwiceyoungerthanF.CishalftheageofAbuttwicetheage
ofD.Whichtwopersonsformthepairofoldestandyoungest?
(a) FandA (b) FandD (c) BandF (d) F andC
Directions(Q.78–82):Readthefollowinginformationcarefullyandanswerthequestionsthatfollow:
P,Q,R,S,TandUaretravellinginabus.Therearetworeporters,twotechnicians,onephotographer
andonewriterinthegroup.ThephotographerPismarriedtoSwhoisareporter.Thewriterismarried
toQwhoisofthesameprofessionasthatofU.P,R,Q,Saretwomarriedcouplesandnobodyinthe
group has same profession. U is brother of R.
Q77. Which of the following is a pair of technicians?
(a)RS (b)SU (c)PT (d)QU
Q82. Four girls are sitting on a bench to be photographed. Shikha is to the left of Reena.
Manju is to the right of Reena. Rita is between Reena and Manju. Who would be
second from the left in the photograph?
(a)Reena (b)Shikha (c)Manju (d)Rita
Q83. Fivechildrenaresittinginarow.SissittingnexttoPbutnotT.KissittingnexttoR
whoissittingontheextremeleftandTisnotsittingnexttoK.Whoaresittingadjacent toS?
(a) KandP (b)RandP (c)OnlyP (d)P andT
Q85. There are five friends. They are standing in a row facing South. Jayesh is to the
immediate right of Alok. Pramod is between Bhagatand Subodh. Subodh is
between Jayesh and Pramod. Who is in themiddle?
(a)Bhagat (b)Jayesh (c)Pramod (d)Subodh
Q88. In which of the following pairs, second person is to the immediate left of the first
person?
(a) „H, E‟
(b)„D, A‟
(c) „B,G‟
(d) „B,C‟
Q89. Which of the following is definitely true?
(a) „D‟issecondtotheleftof„H‟
(b) „A‟ is second to the right of„E‟
(c) „C‟ is to the immediate right of„B‟
(d) „E‟istotheimmediateleftof„C‟
Q97. Fiveboystookpartinarace.RajfinishedbeforeMohitbutbehindGaurav.Ashishfinished
beforeSanchitbutbehindMohit.Whowontherace?
(a)Raj (b)Gaurav (c)Mohit (d)Ashish
Directions(Q.99–104):Studyfollowinginformationcarefullyandanswerthequestions
given below. A, B, C, D, E, F, G and H are eight students of a school. They study in Std. VI,
VII and VIII with not more than three in any Std. Each of them has a favourite subject from
Physics, Geography, English, Marathi, Mathematics, Chemistry, Biology and Economics not
necessarily in the same order.D likes Chemistry and studies in Std. VIII with only H. B does
notstudyinStd.VII.EandAstudyinthesameStd.butnotwithB.CandFstudyinthesame
Std.ThosewhostudyinStd.VIdonotlikeMathematicsorBiology.FlikesPhysics.Theone
Q101. Whichofthefollowingcombinationsofstudent-Std-Subjectiscorrect?
(a)C-VII-Economics (b)D-VI-Chemistry
(c)G-VII-Physics (d) None iscorrect
Syllogism
Directions (Q.104 – 105): In each of the following questions, select one alternative in which
the third statement is implied by the first two statements.
Q103. (a) Allelephantsarewild.Alllionsarewild.So,alllionsareelephants.
(b) Allmangoesarered.Someapplesaremangoes.So,allapplesarered.
(c) All roads are boxes. All foxes are roads. So, all boxes arefoxes.
(d) All XYZ can run. All ABC are XYZ. So, all ABC canrun.
Q106. P: Cups play chess. Chess is a difficult game. Therefore cups play a difficult game.
Q: Ritu is a girl. All girls are timid. ThereforeRitu is timid.
Q107. P: Some musicians are not rich. All musicians are polite. Therefore not all polite
persons are rich.
Q: All musicians are rich. No rich person is polite. Therefore musicians are not
polite.
Directions (Q.109 – 111): In each of these questions, two statements P and Q are given. You
May look into their logical acceptability and mark your answer as
(a) if both P and Q aretrue
(b) If P is correct and Q is wrong ordoubtful
(c) If P is wrong or doubtful and Q iscorrect
(d) IfbothPandQarewrongordoubtful
Q108. P: All A‟s are B‟s. All B‟s are C‟s. Therefore all C‟s are A‟s.
Q: Some A‟s are B‟s. All B‟s are C‟s. Therefore some A‟s are C‟s
Q109. P: Some A‟s and some B‟s are C‟s. Some C‟s are both A‟s and B‟s. Therefore some
Q110. P: AllA‟sareB‟s.SomeB‟sareC‟s.ThereforesomeA‟sareC‟s.
Q: SomeA‟sareB‟s.AllC‟sareA‟s.ThereforesomeC‟sareB‟s.
Directions(Q.112–115):Eachquestiongivenbelowhasasetofthreeorfourstatements.Each
setofstatementsisfurtherdividedintothreesegments.Choosethealternativewherethethird
segment in the statement can be logically deduced using both the preceding two, but not just
from one of them.
Q111. A: X is an actor. Some actors are pretty. X is pretty.
B: Some men are cops. All cops are brave. Some brave people are cops.
C: All actors are brave. Some men are actors. Some men are brave.
D: All actors are pretty. X is not an actor. X is not pretty.
(a) Aonly (b)BandC (c)Conly (d) Donly
Q112. A: All beautiful things are sad. She is beautiful. She is sad.
B: All nice things are flat. TVs are flat. TVs are nice things.
C: Potatoes are stems. All stems are fruits. Potatoes are fruits.
(a) Aonly (b)AandB (c)Conly (d) A andC
Q113. A: All mammals are viviparous. Some fish are viviparous. Some fish aremammals.
B: Allbirdsareoviparous.Somefisharenotoviparous.Somefisharebirds.
C: No mammal is oviparous. Some creatures are oviparous. Some creatures are not
mammals.
D: Somecreaturesaremammals.Somecreaturesareviviparous.Somemammalsare
viviparous.
(a)Aonly (b)Bonly (c)Conly (d) Donly
Q114. A: All good people are knights. All warriors are good people. All knights are
warriors.
Directions(Q.116–121):Eachquestiongivenbelowconsistsoffiveorsixstatementsfollowed
By Options consisting of three statements put together in a specific order. Choose the option
which indicates a valid argument containing logically related statements that is, where the
Third Statement is a conclusion drawn from the preceding two statements.
Q115. A: All synopses are poets. B: Some synopses are mentors.
C: Some X are notmentors. D: All X are poets.
E: All synopses are mentors. F: All synopses are X.
(a)ACB (b)AEC (c) FEC (d)DFA
Directions (Q.122 – 123): Each of these questions is based on a set of given propositions
I to IV.
Selectthebestanswertoeachofthesequestions. I:
All P areQ.
II: Some P are notQ.
III: Some, but not all, P are Q.
IV: No P isQ
Q121. Considered only by themselves, which of the following pairs of statements might
either both be true or both be false?
(a)IandII (b)IIandIII (c)I and III (d)I andIV
Q122. Considered only by themselves, all of the following pairs of statements might both be
false but could not both be true except
(a) IandII (b)IandIII (c)Iand IV (d)II andI
Series Completion
Directions: find the missing term in each of the following series:
Q123. 1,6,15,?,45,66,91
(a)25 (b)26 (c)27 (d)28
Q127. 1, 3, 3, 6, 7, 9, ?, 12, 21
(a)10 (b)11 (c)12 (d)13
357
Q128. Whichfractioncomesnextinthesequence1, , , ,?
2 4 8 16
9 10 11 12
(a) (b) (c) (d)
32 17 34 35
Q133. Which term comes next in the sequence: AC, FH, KM, PR,?
(a)UW (b)VW (c)UX (d)TV
Q134. Find the next term in the series: BMO, EOQ, HQS,?
(a)KSU (b)LMN (c)SOV (d)SOW
Q135. Which term comes next in the series: YEB, WFD, UHG, SKI,?
(a)QOL (b)QGL (c)TOL (d)QNL
Q138. Find the term which does not fit into the series given below:
G4T, J10R , M20P, P43N, S90L,
(a)G4T (b)J10R (c)M20P (d)P43
CODING- DECODING
Directions (Q.140 – 151): In each of the following questions below, find out the correct
Answer from the given alternatives.
Q139. In a certain code, GIGANTIC is written as GIGTANCI. How is MIRACLES written in
that code?
(a)MIRLCAES (b)MIRLACSE (c)RIMCALSE (d)RIMLCAES
Q142. Inacertaincode,RIPPLEiswrittenas613382andLIFEiswrittenas8192.Howis
PILLER written in thatcode?
(a)318826 (b)318286 (c)618826 (d)328816
Q143. If ROSE is coded as 6821, CHAIR is coded as 73456 and PREACH is coded as 961473,
what will be the code for SEARCH?
(a)246173 (b)214673 (c)214763 (d)216473
Q145. In a certain code language, QUEUE is written as Q22, and CHURCH is written as
1UR1. Which of the following would be most appropriate code for BANANA in that
language?
(a)B5A5 (b)5N5A (c) B55A (d)BA5A5A
Q147. If CARPET is coded as TCEAPR, then the code for NATIONAL would be
(a)NLATNOIA (b)LANOITAN (c)LNAANTOI (d)LNOINTAA
Q149. In a code language if POSE is coded as OQNPRTDF, then the word TYPE will be
coded as
(a)SUXZOQFD (b)SUXZQOFD (c)SUXZOQDF (d)SUXZQODE
Q150. Inacertaincode,ORGANISMiswrittenasROAGINMS.HowisBOARDINGwritten in
thatcode?
(a)RAOBIDGN (b)OBRAGNID (c)OBRAIDGN (d)OBIDRAGN
Q151. Inacertaincode,BRIGHTENiswrittenasHJSCMDSG.HowisCOMPLETEwritten in
thatcode?
(a)DSDKQNPD (b)QNPDDSDK (c)QNPDFUFM (d)OLNBFUFM
Q152. InacertaincodeRAINiswrittenas8$%6andMOREiswrittenas7#8@.Howis
REMAIN written in thatcode?
(a)8@7$%6 (b)7@#$%6 (c)#@&$%6 (d)#@7$%
Directions(Q.154–155):Ineachofthefollowingquestionsfindthecorrectanswerfromthe
given alternatives.
Q153. Inacertainlanguage„prenotbis‟means„smokingisharmful‟,„vogdornot‟means
„avoidharmfulhabit‟,and„dorbisyel‟means„pleaseavoidsmoking‟,whichofthe
following means „habit‟ in thatlanguage?
(a)vog (b)not (c)dor (d)bis
Q156. In a queue, Amrita is 10thfrom the front while Mukul is 25thfrom behind and Mamta
is just in the middle of the two. If there be 50 persons in the queue, what position does
Q157. Raman ranks sixteenth from the top and forty ninth from the bottom is a class. How
many students are there in the class?
(a)64 (b)65 (c)66 (d)DataInadequate
Q158. Sanjeevranksseventhfromthetopandtwentyeighthfromthebottominaclass.How
manystudentsarethereintheclass?
(a)37 (b)36 (c)35 (d)34
Q159. If Atul finds that he is twelfth from the right in a line of boys and fourth from theleft,
how many boys should be added to the line such that there are 28 boys in the line?
(a)12 (b)13 (c)14 (d)20
Q160. Manish ranked sixteenth from the top and twenty ninth from the bottom among those
who passed an examination. Six boys did not participate in the competition and five
failed in it. How many boys were there in the class?
(a)40 (b)44 (c)50 (d)55
Q161. Some boys are sitting in a row. P is sitting fourteenth from the left and Q is seventh
fromtheright.IftherearefourboysbetweenPandQ,howmanyboysarethereinthe row?
(a)25 (b)23 (c)21 (d)19
Q165. Inarowoftenboys,whenRohitwasshiftedbytwoplacestowardstheleft,hebecame
seventhfromtheleftend.Whatwashisearlierpositionfromtherightendoftherow? (a)First
(b)Second (c)Fourth (d)Sixth
Q166. In a queue, Vijay is fourteenth from the front and Jack is seventeenth from the end,
while Mary is in between Vijay and Jack. If Vijay be ahead of Jack and there be 48
persons in the queue, how many persons are there between Vijay and Mary?
(a)8 (b)7 (c)6 (d)5
Q167. In a row of girls, Shilpa is 8thfrom left and Reena is 17thfrom the right. If they
Interchange their positions, Shilpa becomes fourteenth from the left. How many girls
are there in the row?
(a) 25
(b) 27
(c) 29
Q168. In a queue of children, Kashish is fifth from the left and Mona is sixth from the right.
When they interchange their places among themselves, Kashish becomes thirteenth
from the left. Then, what will be Mona‟s position from the right?
(a) 4th (b)8th (c)14th (d)15th
Q169. In a row of boys, Kapil is eighth from the right and Nikunj is twelfth from the left.
When Kapil and Nikunj interchange positions, Nikunj becomes twenty first from the
left. Which of the following will be Kapil‟s position from the right?
(a)8th (b)17th (c)21st (d)None ofthese
Q170. Sangeeta remembers that her father‟s birthday was certainly after eighth but before
thirteenth of December. Her sister Natasha remembers that their father‟s birthday was
Q171. Standing on a platform, Amit told Sunita that Aligarh was more than ten kilometres
but less than fifteen kilometres from there. Sunita knew that it was more than twelve
butlessthenfourteenkilometresfromthere.Ifbothofthemwerecorrect,whichofthe
followingcouldbethedistanceofAligarhfromtheplatform?
(a)11km (b)12km (c)13km (d)14km
Q172. ReachingtheplaceofmeetingonTuesday15minutesbefore08.30hours,Anujfound
himself half an hour earlier than the man who was 40 minutes late. What was the
scheduled time of themeeting?
(a)8.00hrs (b)8.05hrs (c)8.15hrs (d)8.45hrs
Q174. ThetrainforLucknowleaveseverytwoandahalffromNewDelhiRailwayStation. An
announcement was made at the station that the train for Lucknow had left 40
minutes ago and the next train will leave at 18.00 hrs. At what time was the
announcementmade?
(a)15.30hrs (b)17.10hrs (c) 16.00 hrs (d)None ofthese
Q177. Statement: The government is soon going to introduce a bill which would permit the
instituting of private universities under very strict direction.
Conclusions:
(a) We have some private universities in our country evennow.
(b) The demand for more universities is being steppedup.
(c) Such directions can also be issued without informing theParliament.
(d) The government gives directions to establish anything in privatesector.
(e) Unless and until the directions are given, the private universities can charge
exorbitantfees.
Q179. Statement: In the university examination, most of the candidates write in Hindi
medium.
Conclusions:
(a) Some candidates of this examination write inHindi.
(b) MostlycandidateswithHindimediumappearinthisexamination.
(c) In this examination no candidate writes answers in medium other thanHindi.
(d) All the candidates who appear in this examination write answers in Hindi.
Q180. Statement: This book can help because all good books help.
Conclusions:
(a)This is not a good book.
(b)This is a good book.
(c)No good book helps.
(d)Some good books help.
Q182. Statement: A forest has as many sandal trees as it has Ashoka trees. Three-fourth of
the trees are old ones and half of the trees are at the flowering stage.
Conclusions:
(a) All Ashoka trees are at the floweringstage.
(b) All sandal trees are at the floweringstage.
(c) Atleastone-halfoftheAshokatreesareold.
Q184. Statement:Inthiscompany,60%oftheemployeesaremales,40%arefemales,80%of
the employees are sincere and 40% of the employees are from this city – Rawalpura.
Conclusions:
(a) All male employees are from outstation.
(b) All male employees aresincere.
(c) 20% of female employees are notsincere.
(d) All female employees are fromRawalpura.
(e) None ofthese
Q185. Statement: In a class, three-fourth of the boys play football, one-half play cricket, one
fourth of those who play cricket do not play football.
Conclusions:
(a) Two-third of the boys play onlyfootball.
(b) One-fourth of the boys play neither cricket norfootball.
(c) One-thirdoftheboysplayneithercricketnorfootball.
(d) One-eighthoftheboysplayneithercricketnorfootball.
(e) Two-fifth of the boys play onlyfootball.
Directions(Q.188–190):Inthefollowingquestionstherearegivensomestatementsfollowed
by conclusions that can be drawn from them. Choose the conclusion which appeals to you to
be the most correct.
Q186. The Taj is in Agra. Agra is in India. Therefore, the Taj is in India.
(a)True (b)Probablyfalse (c)False (d) Can‟tsay
Q188. Hydrogen is lighter than oxygen. Hydrogen is lighter than helium. Therefore, oxygen
is the heaviest of the three gases.
(a) False (b)Probablyfalse (c)True (d) Can‟tsay
Q189. People who are bald are generally of the intellectual type. Arun is bald. Therefore
Arun is an intellectual.
(a)False (b)Probablytrue (c)True (d)Can‟tsay
Directions (Q.192– 196): In each of the following questions, certain statements are given
followed by a conclusion based upon them. Choose the alternative which best applies to the
given statements and conclusion.
Q190. Statements:
1. A triangle has threeangles.
2. A square has fourangles.
Conclusion: A polygon has many angles.
The conclusion drawn is
(a)definitely true
(b)definitely false
(c) either probably true or probablyfalse
(d) irrelevant
Q191. Statements:
1. Some persons are weak inMathematics.
2. Allthose,whoareweakinMathematics,aremusicians.
Conclusion: Some musicians are weak in Mathematics.
The conclusion drawn is
(a)definitelytrue (b)irrelevant
Q193. Statements:
1. Waterboilsat100C.
2. Waterfreezesat0C.
Conclusion: At low pressure, water boils at lower temperatures.
The conclusion drawn is
(a) definitely true (b) definitelyfalse
(c) either probably true orprobablyfalse (d)irrelevant
Q194. Statements:
1. During volcanic eruptions, molten lava oozes out in astream.
2. Thelavacomesfromunderthecrustoftheearth.
Conclusion: The inside of the earth must be very hot.
The conclusion drawn is
(a) definitely true (b) probablytrue
(c) definitelyfalse (d)irrelevant
Q195. Statements:
1. Oxygen is agas.
2. This cylinder containsgas.
Conclusion: This cylinder contains oxygen.
The conclusion drawn is
(a)irrelevant (b)definitelytrue
(THEME DETECTION)
Directions:Eachofthefollowingquestionscontainsasmallparagraphfollowedbyaquestion
on it. Read each paragraph carefully and answer the question given below it:
Q201. The virtue of art does not allow the work to be interfered with or immediately ruled
by anything other than itself. It insists that it alone shall touch the work in order to
Q202. Though the waste of time or the expenditure on fashions is very large, yet fashions
have come to stay. They will not go, come what may. However, what is not required
is that strong efforts should be made to displace the excessive craze for fashion from
the minds of these youngsters.
The passage best supports the statement that:
(a) fashion is the need of theday.
(b) theexcessivecrazeforfashionisdetrimentaltoone‟spersonality.
(c) the hoard for fashion should be done away with so as not to let down the
constructivedevelopment.
(d) workandotheractivitiesshouldbevaluedmorethantheoutwardappearance
Q203. Due to enormous profits involved in smuggling, hundreds of persons have been
attracted towards this anti-national activity. Some of them became millionaires
overnight. India has a vast coastline both on the Eastern and Western Coast. It has
been a heaven for smugglers who have been carrying on their activities with great
impunity. There is no doubt that from time to time certain seizures were made by the
enforcement authorities, during raids and ambush but even allowing these losses the
smugglers made huge profits.
The passage best supports the statement that:
(a) smuggling hampers the economic development of anation.
(b) smuggling ought to becurbed.
Q205. Emerson said that the poet was landlord, sealord, airlord. The flight of imagination
made the poet master of land, sea and air. But a poet‟s dream of yesterday becomes
today an actual achievement and a reality for all men. Even those who invented,
improved and perfected the aeroplane could hardly have dreamt of the possibility of
flight into outer space.
The passage best supports the statement that:
(a) seemingly impossible imaginations make one a goodpoet.
(b) all imaginations become a reality someday.
(c) what man imagined has never been impossible; he has always turned it a reality
through his conception of ideas and sheer hardlabour.
(e)manhasreachedtheclimaxoftechnologicaldevelopmentwithhisexplorationinto
outer space.
Q206. The prevention of accident makes it necessary not only that safety devices be used to
guard exposed machinery but also that mechanics be instructed in safety rules which
they must follow for their own protection, and that lighting in the plant beadequate.
The passage best supports the statement that industrial accidents:
(a) are alwaysavoidable.
(b) may be due toignorance.
Q210. Throughout the ages the businessman has helped build civilisation‟s great cities,
providedpeoplewithluxuriesandartistswithpatronage,andlifthisfellowcitizens
tounderstandthestandardofliving.Inthelastfewcenturiesthebusinessmanhas
seededtheIndustrialRevolutionaroundtheworld.
The passage best supports the statement that the businessman:
(a) is accountable to thesociety.
(b) lives luxurious and comfortablelife.
(c) is the beneficiary of the industrialRevolution.
(d) is capable of raising his standard ofliving.
(e) has contributed to the growth ofcivilization.
Q211. One of the important humanitarian by products of technology is the greater dignity
and value that it imparts to human labour. In a highly industrialized society, there is
no essential difference between Brahmin and Dalit, Muslim and Hindu; they are
equally useful and hence equally valuable, for in the industrial society individual
productivity fixes the size of the pay cheque and this fixes social status.
The passage best supports the statement that:
Q213. The press should not be afraid of upholding and supporting a just and righteous cause.
It should not be afraid of criticizing the government in a healthy manner. The press
has to be eternally vigilant to protect the rights of the workers, backward and
suppressed sections of the society. It should also give a balanced view of the things so
that people can be helped in the formation of a healthy public opinion.
The passage best supports the statement that:
(a) presshasagreatroletoplayinademocracy.
(b) thepressistheonlymeanstoprojecttothemassesthepoliciesofthegovernment.
(c) the freedom of press is essential for the proper functioning ofdemocracy.
(d) thepresscanbeusedbythegovernmentsasaneffectivemediafortheupliftment of the
backward sections ofsociety.
(e) all the information given by the press should be well articulated so as to gain a
good opinion towards the rulingparty
Q214. There is a shift in our economy from a manufacturing to a service orientation. The
increase in service-sector will require the managers to work more with people rather
than with objects and things from the assembly line.
This passage best supports the statement that:
(a) Managers should have a balancedmind.
(b) Assembly line will exist in serviceorganizations.
Q216. America is one country that has no shortage of guns. Almost anyone can lay his
hands on them. It isn't surprising then that the number of gun-related crimes is
always on the rise. Which is why law-abiding citizens must be allowed to keep a gun
on their person or property and therefore protect themselves? Which of the
following statements will weaken theargument?
(a) Using guns for protection is everyone's fundamentalright
(b) Policemen cannot be available whenever a crime isunderway
(c) 55%ofallgunsusedincrimesarestolenfromthehomesoflaw-abidingcitizens
(d) Guns are dangerous regardless of who hasthem
Q217. All over the world, there is a severe shortage of organs for those requiring a
transplant.Afamilymembercandonateanorganonlyifmedicaltestsshowthatthe
organwillbeacceptedbypatient'sbody.Insomecases,thefamilymemberdonates
anorgantosomeoneelse,whoarrangesfortherequiredorgantobedonated;this
can be a complicated chain of donors and organs. In some countries selling organs is
Q218. Since we pay for food and shelter, which are necessary for life, it is perfectly morally
acceptable to pay for water and allow private entities to market it.
Which of the following statements strengthens the argument that water usage should
be payable?
(a) Profit-oriented private sector will ensure regular watersupply.
(b) Water is a renewable resource and need not becommodified.
(c) Water is a basic human right and should not be paid for by the commonman.
(d) Private companies have always wanted to sellwater.
Q219. Banning books that can ruffle minorities is the right approach in a democratic
country like India. Which of the following is an assumption?
(a) Indiaisstillhometoahugesocialistmovement.
(b) India has experienced a violent partition on religiousgrounds
(c) In a democratic set-up, minorities' interests need to be safeguarded.
(d) India has 28 states and 9 unionterritories.
Q221. The rise of inflammatory bowel diseases could be down to our shifting diets causing
a "boom in bad bacteria", according to researchers. The researchers said the high-fat
diet changed the way food was digested and encouraged harmful bacteria.
Microbiologists said modifying gut bacteria might treat the disease.
Which of the following statements best summarizes the proposed treatment for
IBDs?
(a) Reduction in consumption of monosaturated milk fats can ease thepain
(b) A balanced diet comprising fiber and proteins is the bestsolution.
(c) ReshapingthemicrobialbalanceofthegutcouldsolvetheIBDproblem.
(d) Strengtheningthebowelmusclesisthemosteffectivesolution.
Q222. Racist attacks are on the rise in most countries prompting Indian students to do a
rethink on their decision to study abroad. However, governments in the countries
concerned have promised to take up active campaigns to educate the public and
ensure that students are provided a safe and healthy atmosphere to live in. The fact
that hate crimes are on the rise, does not reflect positively on their efforts. The
argument "hate crimes are increasing despite efforts by the authorities" would
weaken if:
(a) Students have to study abroad due to lack of the right teaching facilities inIndia.
(b) The police have reported 30 hate crimes as compared to last year'sfigures.
(c)Racist crimes are now a punishable offence and can lead toimprisonment.
(d) 911 call centers report an 80% reduction in SOS calls received by them in this
category.
Q225. In the early twentieth century, the biggest crisis in America was addiction to liquor.
It was leading to a rise in domestic violence, anger-fuelled crimes, and absenteeism
from work, and pushed society towards self-destruction. Prohibition in the 1930s just
made matters worse, by pushing the business underground, and into the hands of
those who were aggressively pushing for profits, used nefarious means to get their
way, and weren't averse to selling badly brewed liquor, for money. Today, societyis
hurtling towards self-destruction again, and this time the reason is marijuana
Q226. After the huge public rally in support of Hitesh Shah, a businessman and an
independent candidate, whose squeaky clean image is attracting the attention of
voters, the government is in a quandary. The problem was made worse by a media
blitz following an interview with Mr. Shah, who said, 'Before a thief starts stealing,
he gets elected'. This seems to have swung public opinion heavily in his favour, and
the election this time will be a very different affair. Which statement is central to the
argument that Mr. Shah is making?
(a) Politicians are no match for articulatebusinessmen.
(b) The media favours theunderdog.
(c) All politicians arecorrupt.
(d) Theelectionswillbeaffectedbytherecession.
Q227. Huge problems are heading our way. With global warming spiralling out of control
and extreme weather-related phenomena causing unheard of destruction regularly,
we are now at a stage where we have to live to eat. Food is going to become more
and more expensive; we will work in our offices way past our retirement age to be
able to afford all the basic meals.
Which of the following statements is not stated in the passage?
(a) Food is going to become more and moreexpensive
(b) Wewillworkinourofficeswaypastourretirementage.
Q228. Team meetings convened to find solutions to problems are largely silent affairs. But,
brainstorming is the modern manager's answer to getting employees to open up.-
Gather the team in a room, place a whiteboard in front of them, and eventually, after
a series of free associations, the solution everyone is looking for will appear. The
rules are that nobody must scoff at even the most unlikely idea. Which of the
following statements is the underlying assumption of this argument?
(a) Team meetings areuseless
(b) Brainstorming sessions need to bechallenging
(c) If people are scared of saying the wrong thing, they'll hardly openup.
(d) CEOs prefer to think about solutionstogether
Q229. In the last five years, the number of students seeking admission to the government-
funded Godhpur University has been falling by 68%. During a recent budget review,
the state's home minister proposed shutting down the university and using the funds
for other purposes. He has recently shut down two other Universities based on the
minister's claims that the state's funds were being poured into a loss-making
institution. But, even a quick look at the University's distance education programme
shows that the university has about 2.3 lakh students enrolled for various online
courses, this number has been growing steadily for the last five years, filling the
University's coffers.
What can you conclude from the above passage?
(a) MinistersfeelthattheUniversityisadrainoffunds
(b) TherearemorestudentsinGodhpurthaninDelhi
(c) The University has more virtual students than regularones
(d) Offering distance education courses has been a monetary boon for theUniversity
Q230. A new television advert for Diet Bhujia featuring singer D has been cleared by the
Advertising Standards in an Indian state after they received 22complaints.
Q233. Dr. Mira Nair has collected data about student obesity. Her goal is to make students
aware of maintaining healthy eating habits before lifestyle diseases have a chance to
set in. Which of the following statements can be inferred from this passage?
(a) Dr Mira Nair is working towards the immediate well being of theadults.
(b) Dr Mira Nair is working towards the immediate well being of theaged.
(c) DrMiraNairisworkingtowardsthewellbeingofthestudents.
(d) DrMiraNairisworkingtowardsthewellbeingofherself.
Q234. President Vladimir Putin of Russia today signed a new law increasing the fines
against those involved in unlawful protests, overriding concerns from his human
rights advisor and the Council of Europe. The people of Russia believe this law is not
only draconian, but will set Russia apart from the rest of the world by making it look
like a nation still living in the middle ages. The new legislation increases maximum
fines for individuals involved in illegal protests from 100 roubles to 10,000 roubles.
Protestors are banned from concealing their faces and nobody with a criminal record
may organize a protest. Which of the following will weaken the protestor's stand of
'living in the middle ages'?
(a) The government decides to replace a fine with communitywork.
(b) The law matched similar legislation in other Europeannations.
(c) If the protestors manage to get the fine reduced by a thousandroubles.
(d) The Americans decide to campaign against thelaw.
Q238. Teachers are now in touch with their students on Facebook. This now means social
sites will soon be accepted as a compulsory means of communication by all schools.
Soon all students and teachers should compulsorily have an account with Facebook.
Which of the following is analogous to the logic provided above?
(a) Olive oil has been proven to have huge benefits for people suffering from heart
problems. This would mean that soon olive oil would be an integral part of every
doctor's medication list for their heartpatients.
(b) Cyclinghasproventohaveapositiveeffectontheenvironment.Soallcyclists
willsoonhavetheirownfreewayswithplentyofgreencover.
(c) Swimming with dolphins helps children lose their fear of water faster. So all
aquariums will offer swimming classessoon.
(d) A house with pets is more friendly that one with none. So all housingsocieties
will have it as a norm to help foster better relations between inhabitants.
Q239. In a recent survey it was found that 80% of meat eaters lived longer as compared to
vegetarians. Hence it is safe to assume that meat provides us with nutrients that
cannot be replicated by any other food source.
Which of the following is not stated in the passage?
(a) 80% of meat eaters lived longer than non-meat eaters.
(b)A survey was conducted in the not so distant past.
(c) Nutrientsprovidedbyavegetariandietarenotprovidedbyameatdiet.
LOGICAL DEDUCTION
Directions (Q.1 – 11): In each of these questions, there are two statements A and B followed
by two conclusion numbered I and II. Decide which of the two given conclusions logically
follows from the two given statements, disregarding commonly known facts. Mark your
answer as:
Q244. Statements: I. All women are ministers. II. All ministers arefruit.
Conclusions: I. All womenarefruit. II. All ministers arewomen.
Directions(Q.12–24):Ineachofthequestionsbelowaregiventhreestatementsfollowedby
someconclusions.Youhavetotakethegivenstatementstobetrueeveniftheyseemtobeat variance
with commonly known facts. Read all the conclusions and then decide which of the
givenconclusionslogicallyfollowsfromthegivenstatementsdisregardingcommonlyknown
facts.
Q251. Statements:
A. Some chairs areflowers.
B. All flowers aretrees.
C. Some trees areleaves.
Conclusions:
I. some trees are chairs.
II.Some leaves are flowers.
III. No chair is aleaf.
(a)Nonefollows
(b) Only Ifollow
(c) Only IIfollows
(d) Only I and IIIfollow
(e) Allfollow
Q252. Statements:
A. All buildings aremountains.
B. All glasses aremountains.
C. Some mountains arebook.
Conclusions:
Q253. Statements:
A. All trains areroom.
B. No room is abus.
C. All boats arerooms.
Conclusions:
I. No boat isatrain. II. No bus is a boat. III. No train is aroom.
(a) Nonefollows (b) Only I and II follow(c)Only II and III follow
(d) Only I andIIIfollow (e)Only IIfollow
Q254. Statements:
A. Some rings arephones.
B. Some phones arecomputers.
C. Some computers arestations.
Conclusions:
I. Some stationsarerings. II. Some phones arestations.
III. Some computersarerings. IV.Allringsarestations.
(a) Nonefollows (b) Only I and IIfollow
(c)Only I, II andIIIfollow (d) Only II and III follow
(e)Allfollows
Q255. Statements:
A. All needles arethreads.
B. All threads areboxes.
C. All treesboxes.
Conclusions:
I. No needleistree. II. Some trees arethreads.
Q256. Statements:
A. All trees arehills.
B. Some hills areanimals.
C. All animals arebirds.
Conclusions:
I. Some birdsarehills. II. Some birds aretrees.
III. Some animalsaretrees. IV.Noanimalistree.
(a) Nonefollows (b) Only I and IIIfollow
(c) Only I andIVfollow (d) Only either III or IV and Ifollow
(e) None ofthese
Q257. Statements:
A. All lions arejungles.
B. Some jungles arerabbits.
C. All rabbits are elephants.
Conclusions:
I. Some rabbitsarelions. II. Some elephants arejungles.
III. Some elephantsarelions. IV.Someelephantsarerabbits.
(a) Only I andIIIfollow (b) Only I and IIfollow
(c) Only II andIIIfollow (d)OnlyIIIandIVfollow
(e)None ofthese
Q258. Statements:
A. All books arepens.
B. No pens arehouses.
C. All houses aredoors.
Conclusions:
Q259. Statements:
A. Some dogs arerats.
B. All rats aretrees.
C. Some trees are notdogs.
Conclusions:
I. Some treesaredogs. II. All dogs aretrees.
III. All ratsaredogs. IV.Alltreesaredogs.
(a) Nonefollows (b) Only Ifollow
(c) Only I andIIfollow (d) Only II and IIIfollow
(e) Allfollows
Q260. Statements:
A. Some boys arerains.
B. All rains areclouds.
C. Some clouds arecars.
Conclusions:
I. Some cloudsareboys. II. Some cars areboys.
III. Some carsarerains. IV.Somerainsareboys.
(a) Nonefollows (b) Only IVfollows
(c)OnlyIfollow (d) Both I and IVfollow
Q261. Statements:
A. All bricks areflowers.
B. Some houses areflowers.
C. All pens arehouses.
Conclusions:
I. Some housesarebricks. II. Some pens areflowers.
CLASSIFICATION
Directions In each of the following questions, five words are given, out of which four are
same in one way or the other and the fifth one is different from the these four. Select the odd
one.
Q264. (a)Sun (b)Moon (c)Venus (d)Mars
Q291. Three of the following words-form a group, while one does not belong to the group.
Find out that word.
(a)Intestines (b)Eyes (c)Hands (d)Ears
Q292. Four of the following five are alike in a certain way and so form a group. Which is the
one that does not belong to the group?
(a) Volume
(b) Size
(c) Large
Q295. Four of the following five pairs are alike in a certain way and hence form a group.
Which one does not belong to the group?
(a)DONE : NOED
(b)WANT : NATW
(c)WITH : TIHW
(d)HAVE : AVEH
Q296. (a)144
(b)169
(c)256
(d)288
ANALOGIES
Directions: A good way to figure out the relationship in a given question is to make up
asentence that describes the relationship between the first two words. Then, try to use
thesame sentence to find out which of the answer choices completes the same relationship
with
the third word
Q1. Odometer is to mileage as compass is to